Grand Test-21 (Clinical New Pattern)
Grand Test-21 (Clinical New Pattern)
RESULT
Question: 1
A 58-year-old woman with metastatic breast cancer develops exertional
dyspnea and is found to have a large pericardial effusion, jugular venous
distension, and hypotension. Which of the following signs is likely
present?
C Kussmaul sign
Explanation:
https://emedicoz.com/testresult/683510s14692529 1/159
6/9/24, 12:51 PM Neet PG Preparation, Neet PG Coaching, FMGE, USMLE
Correct Answer (B) This is a case of cardiac tamponade due to large pericardial
effusion of metastasis from Carcinoma breast. In these cases, JVP is elevated & it
shows the presence of prominent X descent but Y is obliterated. Kussmaul sign
is never seen in these cases. The specific sign of cardiac tamponade on 2D Echo
is RV diastolic collapse (Inward movement of RV free wall during diastole).
Question: 2
Patient comes palpitation to ER. P-180/min, BP – 70mm of Hg systolic.
ECG shows narrow qrs complex tachycardia with regular heart rate. What
is the next step in management?
A DC Cardioversion
B Adenosine
C IV Fluids
D Inotropes
Explanation:
https://emedicoz.com/testresult/683510s14692529 2/159
6/9/24, 12:51 PM Neet PG Preparation, Neet PG Coaching, FMGE, USMLE
Question: 3
Which of the following is not included in CHADS2Vasc score to assess the
risk of thromboembolism in atrial fibrillation?
A Coagulopathy
B Diabetes
C History of hypertension
Explanation:
Question: 4
Cardioversion is indicated in following arrythmia when the patient
develops hypotension EXCEPT
A PSVT
B Atrial fibrillation
https://emedicoz.com/testresult/683510s14692529 3/159
6/9/24, 12:51 PM Neet PG Preparation, Neet PG Coaching, FMGE, USMLE
D Ventricular tachycardia
Explanation:
Correct Answer (C) Multifocal atrial tachycardia MAT is commonly seen in ICU
patients and is probably due to hyperactivity of the pacemaker cells due to
elevated cytokine levels in the blood. Since the defect is not primarily in the
heart, hypotension doesnot occur due to MAT itself and the treatment is
directed towards management of the underlying cause. Cardioversion is not
indicated.
Question: 5
In a patient with a wide-complex tachycardia, which ONE of the following
ECG findings is not a marker of ventricular tachycardia?
A AV dissociation
B Typical RBBB
C Fusion beats
Explanation:
Correct Answer (B) The signs of Ventricular tachycardia on ECG are – Broad QRS
> 140 msec, AV dissociation, Fusion & capture beats, QRS concordance, Atypical
RBBB/LBBB, Extreme axis deviation.
https://emedicoz.com/testresult/683510s14692529 4/159
6/9/24, 12:51 PM Neet PG Preparation, Neet PG Coaching, FMGE, USMLE
Question: 6
What is the area marked ‘X’
A ST segment
B ST interval
C SU segment
D QT interval
Explanation:
ST interval
https://emedicoz.com/testresult/683510s14692529 5/159
6/9/24, 12:51 PM Neet PG Preparation, Neet PG Coaching, FMGE, USMLE
Question: 7
A 60 yrs. old male with lung cancer complains of right sided chest pain,
tingling and weakness of intrinsic muscles of his right hand as well as
ipsilateral ptosis. Which one of the following complications of lung cancer
is most likely cause?
A Horner’s syndrome
B Superior venacaval
obstruction
C Pancoast syndrome
D Hypertrophic
osteoarthropathy
Explanation:
https://emedicoz.com/testresult/683510s14692529 6/159
6/9/24, 12:51 PM Neet PG Preparation, Neet PG Coaching, FMGE, USMLE
Question: 8
Which of the following drug is known to decrease mortality in critically ill
COVID patient?
A Remdesivir
B Hydroxychloroquine
C Toclizumab
D Dexamethasone
Explanation:
Correct Answer (D) Dexamethasone The RECOVERY trial provides evidence that
treatment with dexamethasone at a dose of 6 mg once daily for up to 10 days
reduces 28-day mortality in patients with Covid-19 who are receiving respiratory
support.
Question: 9
Match column A & B and select answer using the code given below
https://emedicoz.com/testresult/683510s14692529 7/159
6/9/24, 12:51 PM Neet PG Preparation, Neet PG Coaching, FMGE, USMLE
A 2, 3, 1, 5
B 2, 1, 4, 5
C 2, 5, 3, 4
D 4, 1, 3, 5
Explanation:
Correct Answer (B) 2,1,4,5. Asbestosis presents with lower zone involvement.
Caplan syndrome historically first seen in coal workers Mesothelioma most often
presents with a persistent unilateral pleural effusion that may mask the
underlying pleural tumor. However, the pleura may be diffusely thickened. Even
with large effusions, no mediastinal shift is seen on chest radiograph because
the pleural thickening associated with the disease leadsto a fixed chest cavity
size and thoracic restriction. Byssinosis is an occupational lung disease caused by
exposure to cotton dust
Question: 10
Identify the following oxygen delivery device
https://emedicoz.com/testresult/683510s14692529 8/159
6/9/24, 12:51 PM Neet PG Preparation, Neet PG Coaching, FMGE, USMLE
D Venturi mask
Explanation:
Correct Answer (B) High Flow nasal canula (HFNC) High-flow nasal cannula
(HFNC) is a heated and humidified system that allows prescribed fraction of
inspired oxygen (FIO2) levels to be delivered at very high flow rates indicated in
hypoxemic resp failure.
Question: 11
True regarding pleural effusion is/are : A. Normal chest wall expansion on
diseased side B. Stony dullness on percussion C. Coarse Crepitations
heard on auscultation D. Air bronchogram is not the classical radiological
finding. Select the right combination
A If A, B, C are correct
Explanation:
https://emedicoz.com/testresult/683510s14692529 9/159
6/9/24, 12:51 PM Neet PG Preparation, Neet PG Coaching, FMGE, USMLE
Correct Answer (C) B and D are correct statements. In pleural effusion decreased
chest expansion seen on affected side. Breath sounds are diminished. Coarse
crepitations is not a feature of pleural effusion
Question: 12
Match the Following Type
Explanation:
Question: 13
https://emedicoz.com/testresult/683510s14692529 10/159
6/9/24, 12:51 PM Neet PG Preparation, Neet PG Coaching, FMGE, USMLE
A Antiendomysial antibodies
C Anti-saccharomyces
cerevisae antibodies
D Antineutrophil cytoplasmic
antibodies
Explanation:
Correct Answer (A) The findings are suggestive of celiac sprue. Anti endomysial
ab is most specific ab of celiac sprue.
Question: 14
Stool osmotic gap is calculated as
https://emedicoz.com/testresult/683510s14692529 11/159
6/9/24, 12:51 PM Neet PG Preparation, Neet PG Coaching, FMGE, USMLE
A Serum osmolarity – (2 X
[fecal sodium + potassium
concentration])
B Stool osmolarity – (2 X
[fecal sodium + potassium
concentration])
C Serum osmolarity – (2 X
[serum sodium + potassium
concentration])
D 2X(serum osmolarity) –
[fecal sodium + potassium
concentration]
Explanation:
Correct Answer (A) Stool osmotic gap = serum osmolarity – (2 X [fecal sodium +
potassium concentration])
Question: 15
Stool osmotic gap increases and become > 50 mosm/L in all of the
following except
A Lactase deficiency
B Lactulose consumption
C Gluten intolerance
https://emedicoz.com/testresult/683510s14692529 12/159
6/9/24, 12:51 PM Neet PG Preparation, Neet PG Coaching, FMGE, USMLE
Explanation:
Stool osmotic gap increases and become > 50 mosm/L in osmotic diarrhea..
Question: 16
Empty sella syndrome is often characterized by
A Pituitary tumour
B Cretinism
C Acromegaly
D None of these
Explanation:
Correct Answer (D) Empty sella syndrome is due to developmental defect in the
Rathke’s pouch. However, the endocrine cells are present over the floor of the
pituitary fossa and function normally. Hence the person has normal Pituitary
function. It is a Radiological Diagnosis – 1-5 per 1000 population.
https://emedicoz.com/testresult/683510s14692529 13/159
6/9/24, 12:51 PM Neet PG Preparation, Neet PG Coaching, FMGE, USMLE
Question: 17
True regarding pseudohypoparathyroidism include all except:
C Albright dystrophy is
associated with
pseudohypoparathyroidism.
Explanation:
Question: 18
A 21 yr patient presents with complaint of primary amenorrhoea. Her
height is 153cm, weight is 51 kg. She has well developed breast. She has
no pubic hair or axillary hair and hirsuitism. Which of the following is the
most probable diagnosis?
https://emedicoz.com/testresult/683510s14692529 14/159
6/9/24, 12:51 PM Neet PG Preparation, Neet PG Coaching, FMGE, USMLE
A Turner syndrome
D Testicular feminizing
syndrome.
Explanation:
Question: 19
Which of the following approved GLP-1 is an oral agent/s
A EXENATIDE
B LIRAGLUTIDE
C SEMAGLUTIDE
Explanation:
Correct Answer (C) Semaglutide is the first oral GLP-1 receptor agonist approved
for therapy. GLP1 agonists prolong the action of endogenously secreted
https://emedicoz.com/testresult/683510s14692529 15/159
6/9/24, 12:51 PM Neet PG Preparation, Neet PG Coaching, FMGE, USMLE
incretins. It controls sugar and also known to reduce CVS mortality in Type 2 DM
patients. However, it is not used in Type 1 DM.
Question: 20
Brown tumour of the bone is characteristic of:
A Hypothyroidism
B Hyperthyroidism
C Hyperparathyroidism
D Hypoparathyroidism
Explanation:
Hyperparathyroidism causes SALT AND PEPPER SKULL & Brown tumour of bone
due to resorption of trabecular bone
Question: 21
https://emedicoz.com/testresult/683510s14692529 16/159
6/9/24, 12:51 PM Neet PG Preparation, Neet PG Coaching, FMGE, USMLE
A AIDS patients
B Foresters
D Transfusion recipients
Explanation:
Correct Answer (C) Patients without a spleen • Asplenic individuals are more
susceptible to babesiosis
Question: 22
Which of the following causes of CKD doesn’t have enlarged kidneys?
A Early Diabetes
nephropathy.
B Amyloid kidney
D FSGS
Explanation:
https://emedicoz.com/testresult/683510s14692529 17/159
6/9/24, 12:51 PM Neet PG Preparation, Neet PG Coaching, FMGE, USMLE
Question: 23
Differentiating feature of Bartter syndrome versus Gitelman’s is :
A Hyponatremia
B Response to indomethacin
C Hypokalemia
D Metabolic alkalosis
Explanation:
Correct Answer (B) Bartter ‘s is associated with high urinary PGE2 levels and
hence responds to NSAIDS. Gitelman’s has no response to NSAIDS. Both are
characterised by Hypokalemic alkalosis.
Question: 24
A 45 years old Male presented with cough with haemoptysis. He also
complains of low grade fever since last 10 days. On examination there is a
purpuric skin rash over the lower limbs. On investigation Urine revealed
presence of dysmorphic RBC and RBC cast. Serum Creatinine is 3.2. What
is the diagnosis?
A IgA nephropathy
https://emedicoz.com/testresult/683510s14692529 18/159
6/9/24, 12:51 PM Neet PG Preparation, Neet PG Coaching, FMGE, USMLE
B PSGN
C Goodpasture’s syndrome
D Vasculitis syndrome
Explanation:
Correct Answer (D) Vasculitis syndrome In the given case, the patient has GN
(renal involvement) and haemoptysis (lung involvement) indicating pulmonary
renal syndromes. Goodpasture’s syndrome is the classical pulmonary-renal
syndrome with only above 2 organ involvement. Other organ involvement viz.
skin rash, uveitis, etc indicate vasculitis syndromes.
Question: 25
A 25 years old male comes with cough associated with sore throat and
fever since 3 days. He also noticed passing blood in the urine. However
his Serum creatinine is normal and Serum complement levels are normal.
What is the diagnosis?
A IgA nephropathy
B PSGN
C Goodpasture’s syndrome
D Vasculitis syndrome
Explanation:
https://emedicoz.com/testresult/683510s14692529 19/159
6/9/24, 12:51 PM Neet PG Preparation, Neet PG Coaching, FMGE, USMLE
Question: 26
Following an extended course of antibiotics for documented
pneumococcal meningitis, a 14-month old child is noted to have evidence
of a developing hearing impairment. Which of the following would be the
most appropriate testing modality to evaluate the hearing status for this
child?
A Otoacoustic emissions
testing
C Auditory brainstem
responses
D Pneumatoscopy with
tympanograms
Explanation:
https://emedicoz.com/testresult/683510s14692529 20/159
6/9/24, 12:51 PM Neet PG Preparation, Neet PG Coaching, FMGE, USMLE
Question: 27
An 18-month-old boy presents for a follow-up visit 2 weeks after
discharge from the hospital following treatment for Kawasaki disease
with intravenous immunoglobulin (IG) and aspirin. He continues to take
aspirin daily and is scheduled for an echocardiogram in several weeks to
evaluate for coronary artery aneurysm. He is currently not up to date on
his immunizations and is due for several vaccinations. Which of the
following vaccines should be withheld at this time?
B Diphtheria-tetanus-
acellular pertussis (DTaP)
C Pneumococcal conjugate
vaccine (PCV13)
D Haemophilus influenzae
Type b conjugate vaccine (Hib)
Explanation:
Question: 28
A 14-year-old female with cystic fibrosis presents with poor growth and
complaint of increased bruising. She admits that she has been using her
https://emedicoz.com/testresult/683510s14692529 21/159
6/9/24, 12:51 PM Neet PG Preparation, Neet PG Coaching, FMGE, USMLE
A A, B6, D, and K
B A, C, D, and E
C A, D, E, and K
D B1, D, E, and K
Explanation:
Question: 29
A 2-week old infant presents to your office for the first well visit. The
pregnancy and delivery were uneventful, and the infant appears well on
physical examination. The family maintains a strict vegetarian diet, and
the mother is currently breastfeeding. Which of the following do you
prescribe as a supplement for the infant at this time?
A Thiamine
B Selenium
C Vitamin D
https://emedicoz.com/testresult/683510s14692529 22/159
6/9/24, 12:51 PM Neet PG Preparation, Neet PG Coaching, FMGE, USMLE
D Niacin
Explanation:
Correct Answer (C) Vitamin D Per the American Academy of Pediatrics (AAP)
recommendations, all breastfed infants, regardless of maternal diet, require 400
IU/day of vitamin D supplementation. Strict vegetarians who exclude all animal
products from their diets may have breast mild that is deficient in vitamin B and
hence may require supplementation with vitamin B . In addition,
supplementation with iron, zinc, and calcium may is be indicated.
Question: 30
During a well-child examination, an 8-month-old girl is found to have no
change in her weight since her 6-month-old visit. She has fallen from the
25 to the 3 percentile in weight. She was breastfed until 6 months of age
and now takes a soy-protein formula in addition to rice cereal, fruits, and
vegetables. On physical examination, she is noted to have a sharply
demarcated and intensely erythematous rash in the perineal and genital
area, characterized by vesiculobullous and pustular lesions, some of
which have dried to form scaly, eczematoid lesions. A sweat test is
performed, which reveals a sweat chloride level of 75 mEq/L. What is the
most likely cause of this patient’s rash?
D Nutritional vitamin E
deficiency
https://emedicoz.com/testresult/683510s14692529 23/159
6/9/24, 12:51 PM Neet PG Preparation, Neet PG Coaching, FMGE, USMLE
Explanation:
Correct Answer (B) Nutritional zinc deficiency The patient has sweat chloride
results diagnostic of cystic fibrosis. Values > 40 mEq/L are suspicious for the
disorder; values ≥ 60 mEq/L are diagnostic. Pancreatic insufficiency associated
with cystic fibrosis results in malabsorption of fat-soluble vitamins (A, D, E, and
K). Mineral deficiencies, including zinc, are also common. Loss of pancreatic
exocrine function causes inadequate digestion and malabsorption of fats and
proteins. Zinc deficiency causes acrodermatitis enteropathica.
Question: 31
A 12-year-old girl with newly diagnosed systemic lupus erythematosus
(SLE) presents for follow-up. She has been doing fairly well over the last 3
months. Which of the following is associated with a worse prognosis?
B Antinuclear antibodies
(ANA)
C Malar rash
D Arthritis
Explanation:
https://emedicoz.com/testresult/683510s14692529 24/159
6/9/24, 12:51 PM Neet PG Preparation, Neet PG Coaching, FMGE, USMLE
Question: 32
An 8-year-old girl presents with painful swelling along the length of the
left index finger, which is accentuated around the distal interphalangeal
joint; range of motion is limited on physical examination. Nail pitting and
onycholysis are also noted. A radiograph demonstrates soft tissue
swelling and periosteal new bone formation of the left index finger.
History of known trauma is denied. An antinuclear antibody (ANA) test is
strongly positive. Which of the following is most likely to complicate this
patient’s clinical course?
A Uveitis
B Pleuritis
C Labyrinthitis
D Colitis
Explanation:
Correct Answer (A) Uveitis Uveitis is the correct answer. The patient has clinical
and radiographic findings consistent with psoriatic juvenile idiopathic arthritis
(psJIA), which, at onset, typically involves a single or less than a total of 5 joints.
The small joints of the hands, wrist, knee, ankle, or hip are most often affected;
involvement of the distal interphalangeal joint is highly suggestive of psoriatic
arthritis. Associated clinical manifestations of acute or chronic anterior uveitis are
common in girls, especially with ANA positivity (30% to 50% of patients). Regular
ophthalmologic screening is essential to prevent visual loss, and standard JIA
uveitis screening guidelines apply.
Question: 33
A 15-year-old girl with a history of fever, anorexia, myalgias, and fatigue of
3-weeks duration is noted to have scaly erythematous, slightly raised
https://emedicoz.com/testresult/683510s14692529 25/159
6/9/24, 12:51 PM Neet PG Preparation, Neet PG Coaching, FMGE, USMLE
A Chilblains
B Epstein pearls
C Janeway lesions
D Osler nodes
Explanation:
Correct Answer (E) Gottron papules Gottron papules are a relatively common
clinical finding in patients with dermatomyositis, the most common
inflammatory myopathy of childhood and adolescence. They appear as
erythematous, scaly, palpable lesions overlying the metacarpal phalangeal and
intercarpal phalangeal joints. Other cutaneous findings in patients with
dermatomyositis include a characteristic edematous periorbital violaceous
erythema (heliotrope) distributed in a masklike manner, often crossing the nasal
bridge and involving the ears. The rash may also involve the knees, elbows,
medial malleoli, extensor surfaces of the extremities, the trunk, and the buttocks.
Proximal muscle weakness is associated with tenderness on physical examination
and an elevation of muscle-derived enzymes on laboratory testing. Difficulty
with climbing stairs, buttoning, combing hair, and rising from a sitting position
on the floor may all be presenting features or associated symptoms. Both
Janeway lesions and Osler nodes are peripheral cutaneous manifestations of
https://emedicoz.com/testresult/683510s14692529 26/159
6/9/24, 12:51 PM Neet PG Preparation, Neet PG Coaching, FMGE, USMLE
Question: 34
A 6-year-old male is brought to the emergency department because of
abdominal pain, diarrhea, bilateral knee pain with swelling, and purpuric
rash on his legs. These symptoms started approximately 2 weeks after an
upper respiratory infection. Workup reveals leukocytosis and elevated
erythrocyte sedimentation rate. The serum chemistry panel and urinalysis
are normal. What is the best next step in managing this patient?
A Prescribe IV
methylprednisolone.
D Start cyclophosphamide.
Explanation:
Correct Answer (C) Treat with hydration and pain medications. This patient has
Henoch-Schönlein purpura (HSP). Skin, joints, and GI tract are the most
commonly involved organ systems. Most cases occur after a viral infection and
https://emedicoz.com/testresult/683510s14692529 27/159
6/9/24, 12:51 PM Neet PG Preparation, Neet PG Coaching, FMGE, USMLE
resolve with conservative management. Use nonsteroidals for pain control but
avoid if renal disease and significant GI disease are present. Corticosteroids have
been used successfully in those with severe abdominal pain or severe scrotal
swelling/edema.
Question: 35
A 6-year-old boy with a 2-day history of rash presents with the sudden
onset of abdominal pain. The rash consists of ecchymosis, petechiae, and,
in a few areas, palpable purpura. It is located over the lower extremities
and dorsal and plantar surfaces of the feet and buttocks. The abdominal
exam reveals diminished bowel sounds, diffuse tenderness on percussion,
and a palpable sausage-shaped fullness in the right lower quadrant. A
rectal exam reveals currant jelly stool that is guaiac-positive. Which of the
following is the most likely complication associated with this patient’s
abdominal pain?
A Splenomegaly
B Toxic megacolon
C Gastrointestinal reflux
D Volvulus
Explanation:
https://emedicoz.com/testresult/683510s14692529 28/159
6/9/24, 12:51 PM Neet PG Preparation, Neet PG Coaching, FMGE, USMLE
Question: 36
A 45-year-old man comes to the OPD with groin pain and swelling. About a
month ago, he noticed a sore on his penis but did not seek medical
attention because the ulcer was not painful and disappeared within a week.
Several days ago, he began to experience painful swelling in his inguinal
region, with inflammation of the overlying skin and eventual formation of
several draining ulcers. He also has mild fever and malaise that began
around the same time as his groin symptoms. The patient is a sailor. His
other medical problems include well-controlled hypertension and
hyperlipidaemia. Cell scrapings from his lesions show cytoplasmic inclusion
bodies as shown in the image. Which of the following is the most likely
cause of this patient's condition?
A Chlamydia trachomatis
B Haemophilus ducreyi
D Klebsiella granulomatis
Explanation:
https://emedicoz.com/testresult/683510s14692529 29/159
6/9/24, 12:51 PM Neet PG Preparation, Neet PG Coaching, FMGE, USMLE
Question: 37
A 26-year-oId man comes to the clinic due to a 3-day-history of dysuria
and urethral discharge. The symptoms developed about 2 weeks after he
had unprotected sexual intercourse with a new partner. On physical
examination, a mucoid discharge is expressed with gentle milking of the
penis. Gram stain of the discharge reveals numerous neutrophils with
intracellular diplococci. A sample of the discharge is placed on an
antibiotic-containing medium, and bacterial colonies are cultured. Which
of the following terms best describes the medium?
A Enriched
B Enrichment
C Reducing
D Selective
Explanation:
Correct Answer (D) Selective • This patient with dysuria and urethral discharge
has gonococcal urethritis as indicated by the Gram stain showing gram-negative
https://emedicoz.com/testresult/683510s14692529 30/159
6/9/24, 12:51 PM Neet PG Preparation, Neet PG Coaching, FMGE, USMLE
Question: 38
A previously healthy 2-year-old boy is brought to the clinic with fever and
mouth pain that began yesterday. He has consumed an adequate amount
of fluids but refuses to eat due to the pain. The patient has no medical
problems and takes no medications. Physical examination reveals swollen
gums and vesicular inflamed lesions on his hard palate and lips as shown in
the image . He has enlarged and tender cervical lymph nodes. Which of the
following is most likely responsible for this patient's condition?
https://emedicoz.com/testresult/683510s14692529 31/159
6/9/24, 12:51 PM Neet PG Preparation, Neet PG Coaching, FMGE, USMLE
Explanation:
Question: 39
A 35-year-old man comes to the clinic due to 3 months of progressive
swelling under his left jaw. The swelling spontaneously opened and
drained pus a month ago but has not improved or resolved. The patient
has no medical conditions other than dental caries; he had a tooth
extracted prior to symptom onset. Physical examination shows an
indurated, nontender mass in the left submandibular area with a small
opening on the overlying skin. Gentle pressure on the mass yields thick
pus containing sand/grain-like particles. The microbial pathogen with
which of the following characteristics is the most likely cause of this
patient's condition?
A Bipolar-staining, gram-
negative bacilli
B Coagulase-producing,
gram-positive cocci
C Conidia-forming:
dimorphic fungus
https://emedicoz.com/testresult/683510s14692529 32/159
6/9/24, 12:51 PM Neet PG Preparation, Neet PG Coaching, FMGE, USMLE
D Filamentous, branching,
gram-positive bacilli
Explanation:
Question: 40
An 86-year-old woman is hospitalized for a urinary tract infection due to
Escherichia coir and is being treated with ceftriaxone. She has a history of
advanced dementia., coronary artery disease, and congestive heart
failure. On the fifth day of hospitalization, she seems agitated. The nurse
also reports that the patient had 3 episodes of diarrhea the previous
night. Her temperature is 38.30 C. In addition to appropriate hand
hygiene, which of the following equipment is necessary before examining
this patient?
A Facemask, non-sterile
gloves, and gown
https://emedicoz.com/testresult/683510s14692529 33/159
6/9/24, 12:51 PM Neet PG Preparation, Neet PG Coaching, FMGE, USMLE
and gown
Explanation:
Correct Answer (C) Nonsterile gloves and gown • This hospitalized patient's fever
and diarrhea a few days after antibiotic initiation are concerning for C. difficile
infection. • All hospitalized patients require standard precautions, including
handwashing before and after patient contact, proper disposal of cleaning
instruments and linens, and occasional use of gowns and gloves as required eg,
contact with body fluid. • However, cases of suspected or proven C. difficile
infection require additional contact precautions , including handwashing with
soap and water (alcohol-based hand sanitizers do not kill the spores) gown for
any patient contact, and nonsterile gloves that should be changed after contact
with contaminated secretions. • Gloves alone would not be sufficient. In
addition, a dedicated stethoscope and blood pressure cuff should be left in the
patient's room.
Question: 41
Microbiology researchers conduct a series of experiments to determine
how pathogenicity is transmitted among different strains of
Streptococcus pneumoniae. In the first experiment, they inject
nonvirulent strain A into the peritoneal cavity of laboratory mice and
observe no ill effects. In the second experiment, researchers subject
virulent strain B to a detergent agent that kills and lyses the bacterial
cells. They then inject the lysate into the peritoneal cavity of a new group
of mice and again observe no ill effects. During a third experiment they
inject live strain A bacteria in combination with the killed strain B lysate;
resulting in death of the mice. Which of the following genetic processes
most likely accounts for the observed findings of these experiments?
https://emedicoz.com/testresult/683510s14692529 34/159
6/9/24, 12:51 PM Neet PG Preparation, Neet PG Coaching, FMGE, USMLE
A Direct uptake of
extracellular DNA
B Phage-mediated DNA
transfer
C Pilus-mediated DNA
transfer
D Spontaneous DNA
mutation
Explanation:
Correct Answer (A) Direct uptake of extracellular DNA • It’s a gene transfer
method named as Transduction. • Certain strains of Streptococcus pneumoniae
express capsular polysaccharides that inhibit phagocytosis, making it a
successful pathogen. • Strains lacking the capsule are not pathogenic: however,
S. pneumoniae is able to obtain new genetic material from the environment that
is released following the death and lysis of neighbouring bacterial cells. • This
process. known as transformation, allows the bacterium to take up exogenous
DNA fragments, integrate the DNA into its genome, and express the encoded
proteins. • Through this method, nonvirulent strains of S. pneumoniae that do
not form a capsule can acquire the genes that code for the capsule and
therefore gain virulence. • Bacteria that have the innate capacity to undergo
transformation are said to be naturally competent and include Haemophilus,
Streptococcus, Bacillus. and Neisseria species.
Question: 42
A 40-year-old farmer from Ohio seeks evaluation at a clinic with complaints
of achronic cough, fevers, and anorexia of several months duration. On
examination, he has generalized lymphadenopathy with
hepatosplenomegaly. A chest radiograph reveals local infiltrates and patchy
opacities involving all lung fields. Fine needle aspiration of an enlarged
lymph node shows the presence of intracellular yeast. Microscopy from
https://emedicoz.com/testresult/683510s14692529 35/159
6/9/24, 12:51 PM Neet PG Preparation, Neet PG Coaching, FMGE, USMLE
fungal culture shown in the image. Which of the following is the most likely
diagnosis?
A Blastomycosis
B Histoplasmosis
C Cryptococcosis
D Sporotrichosis
Explanation:
Correct Answer (B) Histoplasmosis • Of the listed signs and symptoms, the
patient is most likely to have histoplasmosis. • Histoplasmosis is caused by
spores generally found in bird droppings. • Histoplasmosis presents as a cough.
fevers. Anemia, lymphadenopathy, and hepatosplenomegaly with a high fatality
rate. • A lymph node aspiration showed the presence of intracellular yeast, which
facilitates the diagnosis of fungi. • Thick-walled spherical spores with tubercles
and microconidia are typical diagnostic features of histoplasmosis.
Question: 43
A 65-year-old woman in suffers from a macrocytic anemia. Her history is
significant for frequent consumption of pickled fish. A stool sample reveals
https://emedicoz.com/testresult/683510s14692529 36/159
6/9/24, 12:51 PM Neet PG Preparation, Neet PG Coaching, FMGE, USMLE
parasite ova like the one shown in the image. Upon treatment, she expels a
tapeworm in stool. With which tapeworm was she most likely infected?
A Diphyllobothrium latum
B Echinococcus granulosus
C Hymenolepis nana
D Taenia saginata
Explanation:
Correct Answer (A) Diphyllobothrium latum • This cestode, sometimes called the
broad fish tapeworm, has the tendency to absorb large quantities of vitamin B12
from the intestinal tract of the human host—thus producing a tapeworm
pernicious anemia. • Freshwater fish carry infective juvenile tapeworms, but
cooking the fish will inactivate these encysted juveniles. The egg—noted in the
stool concentration—features an an opercular knob, and the scolex is adorned
with bothria and not suckers or hooks. • These are considered reliable markers in
the identification of the species in the clinical laboratory
Question: 44
https://emedicoz.com/testresult/683510s14692529 37/159
6/9/24, 12:51 PM Neet PG Preparation, Neet PG Coaching, FMGE, USMLE
B Submit to a thorough
screening for common sexually
transmitted infections
Explanation:
Question: 45
Which cytokine is essential for T-cell proliferation and is also necessary
for the production of CD25-positive regulatory T cells?
A IL-2
B IL-3
C IL-4
D IL-6
Explanation:
Question: 46
What is the treatment of stage 4 sleep disorders or parasomnias?
A Melatonin
https://emedicoz.com/testresult/683510s14692529 39/159
6/9/24, 12:51 PM Neet PG Preparation, Neet PG Coaching, FMGE, USMLE
B Ramelteon
C Zolpidem
D Benzodiazepines
Explanation:
Question: 47
REM sleep behavioural disorder is mostly seen in association with?
(NIMHANS 19)
D Semantic variant of
frontotemporal dementia
Explanation:
Correct Answer (A) Lewy body dementia “REM behaviour disorder (RBD) involves
a failure of the patient to have atonia (sleep paralysis) during the REM stage
sleep. The result is that the patient literally enacts his or her dreams. Under
normal circumstances, the dreamer is immobilized by REM-related
hypopolarization of alpha and gamma motor neurons. Without this paralysis or
https://emedicoz.com/testresult/683510s14692529 40/159
6/9/24, 12:51 PM Neet PG Preparation, Neet PG Coaching, FMGE, USMLE
with intermittent atonia, punching, kicking, leaping, and running from bed
during attempted dream enactment occur. The activity has been correlated with
dream imagery, and, unlike during sleepwalking, the individual seems unaware
of the actual environment but rather is acting on the dream sensorium. Thus, a
sleepwalker may calmly go to a bedroom window, open it, and step out. By
contrast, a person with REM sleep behavior disorder would more likely dive
through the window thinking it is a dream-visualized lake. Patients and bed
partners frequently sustain injury, which is sometimes serious (e.g., lacerations,
fractures). A wide variety of drugs and comorbid conditions can precipitate or
worsen RBD. In animals, presumed RBD can be produced with bilateral peri-
locus coeruleus lesions. In humans, there is a suggestion that RBD may result
from diffuse hemispheric lesions, bilateral thalamic abnormalities, or brainstem
lesions. Clonazepam has been used successfully to treat RBD.”
Question: 48
The parents of a 10-year-old child report that the child is still wetting the
bed. The child is very upset about this because he would like to go away
to summer camp but is afraid that he will wet the bed there as well.
Physical examination is unremarkable and the child is otherwise
developing typically for his age. Behavioral interventions such as limiting
fluids before bed and the bell and pad apparatus have not been effective.
At this time, which of the following is the best choice for pharmacologic
management of enuresis in this child
A Imipramine
B Diazepam
C Desmopressin acetate
D Acetaminophen
Explanation:
https://emedicoz.com/testresult/683510s14692529 41/159
6/9/24, 12:51 PM Neet PG Preparation, Neet PG Coaching, FMGE, USMLE
Correct Answer (C) Desmopressin acetate The best choice for the pharmacologic
management of bedwetting in an older child such as this is desmopressin
acetate. Imipramine is also useful in managing enuresis but has more side
effects.
Question: 49
A childhood friend makes a visit to your office. The last time you saw her
was at her wedding one year ago. As the conversation proceeds, you ask
her casually if married life is treating her well. She suddenly bursts into
tears and admits that her husband is physically abusive, but that she can't
leave him because she is deeply afraid of being alone. Going back to
childhood, this woman has always had her parents make decisions for her,
including those regarding her education and career. She has always been
very pleasant and courteous in her dealings with others, and avoids
arguments or disagreements whenever possible. Which of the following
personality disorders does this woman most likely suffer from?
A Schizoid personality
disorder
B Dependent personality
disorder
C Schizotypal personality
disorder
D Avoidant personality
disorder
Explanation:
https://emedicoz.com/testresult/683510s14692529 42/159
6/9/24, 12:51 PM Neet PG Preparation, Neet PG Coaching, FMGE, USMLE
and submissive, and crave the protection and care of others. They avoid taking
the initiative because of feelings of inadequacy. (Choice A) Schizoid personality
disorder is characterized by an inability to express emotion and social
detachment. These patients do not enjoy close relationships and prefer to be
aloof and isolated. They rarely indulge in any pleasurable activities and appear
indifferent to praise or criticism from others. (Choice C) Schizotypal personality
disorder is characterized by a pattern of odd and eccentric behavior and a
reduced capacity for close relationships. These patients usually exhibit "magical
thinking." They may have bizarre fantasies or believe in telepathy, clairvoyance,
or the concept of a sixth sense. They often have paranoid ideation and unusual
perceptual experiences. (Choice D) Avoidant personality disorder is characterized
by hypersensitivity to criticism, social inhibition, and feelings of inadequacy.
Although these individuals want friendships, they avoid (as the name suggests)
such intimate relationships because they fear ridicule. They also perceive
themselves as inferior to others and are reluctant to engage in new activities or
to take risks for fear of being embarrassed. Although they appear shy,
individuals with this condition actually want to make friends.
Question: 50
A young girl with anorexia nervosa is on treatment. Even after taking
adequate food according to the recommended diet plan for last 1 week,
there is no gain of weight. What is the next step in management? (AIIMS
NOV 19)
Explanation:
Correct Answer (B) Observe the patient for 2 hours after giving meal (Ref :
Kaplan) “The following considerations apply to the general management of
patients with anorexia nervosa during a hospitalized treatment program. •
Patients should be weighed daily, early in the morning after emptying the
bladder. • The daily fluid intake and urine output should be recorded. • If
vomiting is occurring, hospital staff members must monitor serum electrolyte
levels regularly and watch for the development of hypokalemia. • Because food
is often regurgitated after meals, the staff may be able to control vomiting by
making the bathroom inaccessible for at least 2 hours after meals or by having
an attendant in the bathroom to prevent the opportunity for vomiting. •
Constipation in these patients is relieved when they begin to eat normally. •
Stool softeners may occasionally be given, but never laxatives. • If diarrhea
occurs, it usually means that patients are surreptitiously taking laxatives. •
Because of the rare complication of stomach dilation and the possibility of
circulatory overload when patients immediately start eating an enormous
number of calories, the hospital staff should give patients about 500 calories
over the amount required to maintain their present weight (usually 1,500 to
2,000 calories a day). • It is wise to give these calories in six equal feedings
throughout the day, so that patients need not eat a large amount of food at one
sitting.”
Question: 51
From what distance can a 6/6 patient can see the 6/24 letter on Snellen
chart:
A 60 m
B 18 m
C 24 m
D 6m
Explanation:
https://emedicoz.com/testresult/683510s14692529 44/159
6/9/24, 12:51 PM Neet PG Preparation, Neet PG Coaching, FMGE, USMLE
Correct Answer (C) 6/6 person can see 6/24 letter from 24 m
Question: 52
True about pigmentary glaucoma:
B More common in
hypermetropes
C Slit-like transillumination
defects in the mid periphery
Explanation:
Correct Answer (C) Iris transillumination defects are seen More common in
myopic young males
Question: 53
Which extraocular muscle causes pure depression in adduction?
A Inferior rectus
B Inferior oblique
C Superior rectus
https://emedicoz.com/testresult/683510s14692529 45/159
6/9/24, 12:51 PM Neet PG Preparation, Neet PG Coaching, FMGE, USMLE
D Superior oblique
Explanation:
Question: 54
The following is sign is seen in which retinal layer?
A A
B B
C C
D D
Explanation:
https://emedicoz.com/testresult/683510s14692529 46/159
6/9/24, 12:51 PM Neet PG Preparation, Neet PG Coaching, FMGE, USMLE
Question: 55
Identify the fluoroscein angiography image given for a diabetic female
B Microaneurysms
C Stargardts disease
Explanation:
Question: 56
A 50 year old patient has been wearing – 2 D glasses for both eyes. He
noticed blurring of vision in his right eye recently and went to the doctor.
To his surprise, the doctor told that his power of right eye has increased
to -5 D while left eye was still the same. Vision in both eyes was 6/6 with
full correction. What could be the possibility?
https://emedicoz.com/testresult/683510s14692529 47/159
6/9/24, 12:51 PM Neet PG Preparation, Neet PG Coaching, FMGE, USMLE
A Nuclear cataract
B Nuclear sclerosis
C Posterior subcapsular
cataract
D Keratoconus
Explanation:
Correct Answer (B) Nuclear cataract and PSC will not have 6/6 vision
Keratoconus is in teenage
Question: 57
If caloric testing is done in this patient, what can be true for this patient?
https://emedicoz.com/testresult/683510s14692529 48/159
6/9/24, 12:51 PM Neet PG Preparation, Neet PG Coaching, FMGE, USMLE
Explanation:
Correct Answer (A) This is right fast phase Right nystagmus COWS Cold water in
left ear induce right nystagmus
Question: 58
A 64-year-old man comes to the emergency department due to sudden
painless loss of vision in the right eye. He has a history of hypertension
and type 2 diabetes mellitus. He has had no trauma but gave history of
similar episode in the past. Cardiovascular examination reveals irregularly
irregular rhythm with no murmurs. Visual acuity in the right eye is hand
motion only and normal in the left eye. Fundoscopic examination
revealed edema of the macular leaving the foveal area. The pathology can
be most likely attributed to?
A Retinal arteries
B Retinal veins
C Retinal photoreceptors
D Optic disc
Explanation:
https://emedicoz.com/testresult/683510s14692529 49/159
6/9/24, 12:51 PM Neet PG Preparation, Neet PG Coaching, FMGE, USMLE
Correct Answer (A) Central retinal artery occlusion leading to cherry red spot
Question: 59
Bitemporal superior quadrantanopia is seen in lesion of?
A Pituitary adenoma
B Craniopharyngioma
C Genu of Willebrand
D Tuberculum sellae
meningiomas
Explanation:
Question: 60
The given chart checks which sense?
https://emedicoz.com/testresult/683510s14692529 50/159
6/9/24, 12:51 PM Neet PG Preparation, Neet PG Coaching, FMGE, USMLE
A Form
B Contrast
C Colour
D Motion
Explanation:
Question: 61
Which of the following is a true statement :
A Reservoir of plague :
domestic rats
Explanation:
https://emedicoz.com/testresult/683510s14692529 51/159
6/9/24, 12:51 PM Neet PG Preparation, Neet PG Coaching, FMGE, USMLE
Correct Answer (C) • Reservoir of plague : wild rodents • Scratches with oozing
of blood is classified under Cat 3 exposure • Yersinia pestis is category A
bioterrorism agent • Vector of scrub typhus : Soft tick
Question: 62
Indicator used to identify high risk age groups for mortality :
Explanation:
Question: 63
A pregnant was diagnose to be HIV positive in second trimester. What's
next line of management:
https://emedicoz.com/testresult/683510s14692529 52/159
6/9/24, 12:51 PM Neet PG Preparation, Neet PG Coaching, FMGE, USMLE
Explanation:
Correct Answer (B) With effect from 1st January 2014, pregnant women who are
found to be HIV positive are initiated on lifelong ART irrespective of CD4 count
Question: 64
Which of the false regarding AFP Surveillance :
Explanation:
Correct Answer (D) • Non-polio AFP rate is an indicator for operational efficiency
• Samples should reach laboratory within 72 hrs from the point of collection •
https://emedicoz.com/testresult/683510s14692529 53/159
6/9/24, 12:51 PM Neet PG Preparation, Neet PG Coaching, FMGE, USMLE
Samples are transported at 2-8 degrees • Adequate stool sample collection from
80% of cases
Question: 65
A chest physician observed that the distribution of forced expiratory
volume (FEV) in 200 smokers had a median value of 3 litres with the first
and third quartiles being 1.5 and 4.5 litres respectively. Based on this data
how many persons in the sample are expected to have FEV below 4.5
litres?
A 50
B 100
C 25
D 150
Explanation:
https://emedicoz.com/testresult/683510s14692529 54/159
6/9/24, 12:51 PM Neet PG Preparation, Neet PG Coaching, FMGE, USMLE
Question: 66
Which of the following is true statement as per the information given below
in the table :
Explanation:
Correct Answer (D) • The study is retrospective • Odds ratio will be more than 1 .
OR = 9 • Drug abuse is risk factor for stroke • Chi square test can be applied
here to assess statistical significance of association
Question: 67
https://emedicoz.com/testresult/683510s14692529 55/159
6/9/24, 12:51 PM Neet PG Preparation, Neet PG Coaching, FMGE, USMLE
The study included 1,000 who took the new drug for 5 years and 1,000
were given the standard therapy. At the end of the trial, 6% of the men in
the standard therapy group experienced a stroke, compared to only 2% in
the group taking the new drug. Which of the following is true :
C NNT is 20
D None of these
Explanation:
Correct Answer (A) Absolute risk reduction : 6-2 = 4% NNT = 100/4 = 25 So,25
men had to receive the new drug for 5 years in order for one man to benefit (i.e.
one less stroke to occur)
Question: 68
Which of the following are deficiencies screened under RBSK :
2. Vitamin A deficiency
3. Vitamin D deficiency
4. Zinc deficiency
https://emedicoz.com/testresult/683510s14692529 56/159
6/9/24, 12:51 PM Neet PG Preparation, Neet PG Coaching, FMGE, USMLE
A 1 only
B 1,2
C 1,2,3
D 1,2,3,4
Explanation:
Question: 69
A prospective study is designed to assess the association between arsenic
and the risk of cancer. Small studies conducted earlier suggest a possible
association. This study intends to find the significant association. Which
of the following is most important to not commit type 2 error ?
https://emedicoz.com/testresult/683510s14692529 57/159
6/9/24, 12:51 PM Neet PG Preparation, Neet PG Coaching, FMGE, USMLE
Explanation:
Question: 70
You are evaluating a journal article describing a test for the diagnosis of
congestive heart failure (CHF). In the study described, 250 consecutive
patients were given the test. Of the 250 subjects, 100 tested positive tor
CHF and 150 tested negative. All 250 subjects were then evaluated by
expert cardiologists who were blinded to the results at the experimental
test. These cardiologists determined that out of persons who tested
positive, 90 actually had CHF. Further, the cardiologists found that of the
150 who tested negative, 20 truly had CHF. What is the positive predictive
value (PPV) of this test for the diagnosis of CHF?
A 60%
https://emedicoz.com/testresult/683510s14692529 58/159
6/9/24, 12:51 PM Neet PG Preparation, Neet PG Coaching, FMGE, USMLE
B 70%
C 80%
D 90%
Explanation:
Question: 71
Based on the given data , calculate IMR :
• Population : 10000
• Neonatal deaths : 10
https://emedicoz.com/testresult/683510s14692529 59/159
6/9/24, 12:51 PM Neet PG Preparation, Neet PG Coaching, FMGE, USMLE
Explanation:
Correct Answer (B) • Population : 10000 • CBR : 20 per 1000 • Neonatal deaths :
10 • Post neonatal deaths : 2 So, Number of live births : 200 Infant deaths : 12
IMR = 12 / 200 = 60 per 1000 live births
Question: 72
Which of the following is not a target of reduction in Poshan Abhiyan :
A Anaemia by 3% annually
B Under-nutrition by 2%
annually.
C LBW by 2% annually
D Stunting by 5% annually
Explanation:
https://emedicoz.com/testresult/683510s14692529 60/159
6/9/24, 12:51 PM Neet PG Preparation, Neet PG Coaching, FMGE, USMLE
- Implimented in 2017
Targets : To reduce -
- Anaemia by 3% annually
- Under-nutrition by 2% annually.
- LBW by 2% annually
- Stunting by 2% annually
Question: 73
Which of the following is not a target of “Global action plan for NCDs” :
Explanation:
https://emedicoz.com/testresult/683510s14692529 61/159
6/9/24, 12:51 PM Neet PG Preparation, Neet PG Coaching, FMGE, USMLE
Correct Answer (C) “Global action plan for NCDs” : 9 targets to be achieved by
2025 • A 25% relative reduction in the overall mortality from cardiovascular
diseases, cancer, diabetes, or chronic respiratory diseases : (25 by 25 – i.e To be
achieved by 2025) • At least 10% relative reduction in the harmful use of alcohol
• A 10% relative reduction in prevalence of insufficient physical activity • A 30%
relative reduction in mean population intake of salt/sodium • A 30% relative
reduction in prevalence of current tobacco use in aged 15+ years • A 25%
relative reduction in the prevalence of raised blood pressure • Halt the rise in
diabetes and obesity • At least 50% of eligible people receive drug therapy and
counselling (including glycaemic control) to prevent heart attacks and strokes •
An 80% availability of the affordable basic technologies and essential medicines,
including generics, required to treat major NCDs in both public and private
facilities
Question: 74
Which of the following protocol is used to assess risk factors for NCDs :
A GATHER approach
B STEPS protocol
C SPIKES protocol
D Helsinki declaration
Explanation:
https://emedicoz.com/testresult/683510s14692529 62/159
6/9/24, 12:51 PM Neet PG Preparation, Neet PG Coaching, FMGE, USMLE
Question: 75
Which of the following is true regarding this scheme :
D None of these
Explanation:
https://emedicoz.com/testresult/683510s14692529 63/159
6/9/24, 12:51 PM Neet PG Preparation, Neet PG Coaching, FMGE, USMLE
Question: 76
In a case of chest injury which of the following is not included in lethal six
of deadly dozen?
A Tension pneumothorax.
B Massive hemothorax.
C Pericardial tamponade.
D Aortic disruption.
Explanation:
https://emedicoz.com/testresult/683510s14692529 64/159
6/9/24, 12:51 PM Neet PG Preparation, Neet PG Coaching, FMGE, USMLE
Question: 77
Which of the following is a feature of Karnohan’s notch phenomenon?
B Contralateral pupillary
dilatation
C Ipsilateral Hemiparesis
D Contralateral Hemiparesis
Explanation:
https://emedicoz.com/testresult/683510s14692529 65/159
6/9/24, 12:51 PM Neet PG Preparation, Neet PG Coaching, FMGE, USMLE
Diffusion tensor imaging (DTI) and transcranial magnetic stimulation (TMS) could
be useful for exploring the state of the corticospinal tract (CST).
Question: 78
A man presented with fractures of 4th to 10th ribs and respiratory
distress after a Road Traffic Accident (RTA). He is diagnosed to have
flail chest and a PaO2 of <60 mm Hg, management is:
A Tracheostomy
C Fixation of ribs
D Strapping of chest
Explanation:
Correct Answer (B) Flail chest with PaO2 < 600 mmHg should be treated by PPV
Question: 79
Most common site of subependymal giant cell astrocytoma
https://emedicoz.com/testresult/683510s14692529 66/159
6/9/24, 12:51 PM Neet PG Preparation, Neet PG Coaching, FMGE, USMLE
B Foramen of monro
C 4th Ventricle
D Lateral ventricles
Explanation:
Correct Answer (B) Subependymal giant cell astrocytomas are believed to arise
from a subependymal nodule present in the ventricular wall in a patient with
tuberous sclerosis that occurs in the wall of the lateral ventricle and foramen of
Monro and, rarely, in the third ventricle. Histologically, subependymal nodules
and subependymal giant cell tumors are essentially indistinguishable. The cells
that appear astrocytic, usually resemble gemistocytes (large polygonal cells with
prominent eosinophilic cytoplasm) with a smaller number of ganglionic
appearing giant pyramidal-like cells. Current evidence suggests that they are of
a mixed neuronal and glial lineage, although they continue to be classified as
astrocytomas 5. Subependymal giant cell astrocytomas are- • S100: positive •
GFAP: variable/ synaptophysin: variable • CD34: negative • Additional variable
and focal reactivity: class III beta-tubulin, NeuN, SOX2
Question: 80
Which of the following is not a class I Ag (HLA)?
A HLA A
B HLA B.
C HLA C.
https://emedicoz.com/testresult/683510s14692529 67/159
6/9/24, 12:51 PM Neet PG Preparation, Neet PG Coaching, FMGE, USMLE
D HLA DR.
Explanation:
Correct Answer (D) The major histocompatability antigen are the glycoprotein
on the cell membrane and are encoded by the major histocompatibility complex
(MHC) gene present on the short arm of the Chr. 6. These agents are divided
into class I and II. Class I antigen are HLA-A, HLA-B, and HLA-C, present on all
nucleated cells and are detected by serotyping T Lymphocytes. Class I antigen
are HLA-DR, HLA-DQ, and HLA-DP. Present on B lymphocytes activated T Cells,
Monocytes, Macrophages, dendritic cells and some endothelial cells and HLA-
DR is detected serotyping B lymphocytes (Testing for HLA-DQ and DP is not
routinely done).
Question: 81
Which organ is least commonly involved in acute graft versus host
disease?
A Lung
B Skin
C Liver
D Intestine
Explanation:
Correct Answer (A) Graft-versus-host disease (GVHD) occurs due to the presence
of immunocompetent T lymphocytes in the graft attacking the immunodeficient
recipient tissue due to histocompatibility differences within 100 days, causing
tissue damage Graft-versus-host disease (GvHD) is a systemic disorder that
occurs when the graft's immune cells recognize the host as foreign and attack
https://emedicoz.com/testresult/683510s14692529 68/159
6/9/24, 12:51 PM Neet PG Preparation, Neet PG Coaching, FMGE, USMLE
the recipient’s body cells. “Graft” refers to transplanted, or donated tissue, and
“host” refers to the tissues of the recipient. It is a common complication after
allogeneic hematopoietic stem cell transplant (HCT) GvHD occurs in the
following settings: • Following allogeneic bone transplantation (most common) •
Following transplantation of solid organs that are rich in lymphoid cells (eg.
Intestinal and liver transplant) • Following transfusion of un-irradiated blood
Steroid/ Cyclosporine and tacrolimus/ sirolimus, pentostatin, etanercept, and
alemtuzumab. In August 2017 the US FDA approved ibrutinib to treat chronic
GvHD after failure of one or more other systemic treatments
Question: 82
A patient on TPN developed dry, scaly, pruritic rash on the trunk and
extremities. This lesion is most likely caused by:
A Zinc deficiency
B Vitamin A deficiency
C Vitamin C deficiency
Explanation:
https://emedicoz.com/testresult/683510s14692529 69/159
6/9/24, 12:51 PM Neet PG Preparation, Neet PG Coaching, FMGE, USMLE
Question: 83
During a diaper change in a six week old child her mother noted a lesion in
the umbilicus. The lesion was not noted during a previous office visit when
the infant was 10 days old and still had the umbilical cord attached. On
examination, there was a soft, red mass at the base of the umbilicus.
Although the lesion was slightly moist, no foul discharge or purulent
drainage was present. The surrounding skin was not red, warm, or swollen.
Which one of the following is the most likely diagnosis?
A Umbilical hernia.
B Umbilical pyogenic
https://emedicoz.com/testresult/683510s14692529 70/159
6/9/24, 12:51 PM Neet PG Preparation, Neet PG Coaching, FMGE, USMLE
granuloma.
C Omphalitis.
D Patent urachus.
Explanation:
Correct Answer (B) Umbilical pyogenic granuloma. A solid, red umbilical mass
having a soft, velvety appearance without a fistulous tract suggests a granuloma.
An umbilical pyogenic granuloma is a common, benign abnormality in neonates.
It develops within the first few weeks of life and should not be present at birth. It
forms from excess granulation tissue persisting at the base of the umbilicus after
cord separation. Umbilical granulomas often appear pedunculated and can
secrete a small amount of serous or serosanguineous drainage. Conventional
management has been to dry the umbilical stump and carefully cauterize the
granuloma with a 75 percent silver nitrate stick.
Question: 84
Which of the following statements regarding unusual hernias is incorrect?
https://emedicoz.com/testresult/683510s14692529 71/159
6/9/24, 12:51 PM Neet PG Preparation, Neet PG Coaching, FMGE, USMLE
Explanation:
Correct Answer (C) Sciatic hernias usually present with intestinal obstruction or a
mass in the gluteal or infra-gluteal region.
Question: 85
Which alpha-blocker has the strongest association with floppy iris
syndrome?
A Alfuzosin
B Prazosin
C Tamsulosin
D Doxazosin
Explanation:
Correct Answer (D) Intraoperative floppy iris syndrome (IFIS) is a clinical entity
originally described by Chang and Campbell is associated maximally with
Tamsulocin. This syndrome is known to cause a poor pupillary dilation,
intraoperative complications during cataract surgery and is characterized by the
following triad of intraoperative signs: • Billowing of a flaccid iris stroma in
response to ordinary intraocular fluid currents; • Propensity for iris prolapse
toward the phaco and/or the side-port incisions; • Progressive pupil constriction.
https://emedicoz.com/testresult/683510s14692529 72/159
6/9/24, 12:51 PM Neet PG Preparation, Neet PG Coaching, FMGE, USMLE
Question: 86
SURGERY In penile fracture when Tunica albuginea damaged but Bucks
Fascia is intact. This suggests hematoma in?
A Butterfly hematoma
perineum
D Penile + scrotum
Explanation:
Intact buck’s fascia ensures that bleeding will remain confined to shaft only.
Question: 87
Histologically Zellballen pattern is seen in?
https://emedicoz.com/testresult/683510s14692529 73/159
6/9/24, 12:51 PM Neet PG Preparation, Neet PG Coaching, FMGE, USMLE
D Tumour of Medulla of
adrenal
Explanation:
Tumor cell nest are of chromaffin cells or chief cells with pale eosinophilic
staining. separated into groups by segmenting bands of fibrovascular stroma
and are surrounded by supporting sustentacular cells.
https://emedicoz.com/testresult/683510s14692529 74/159
6/9/24, 12:51 PM Neet PG Preparation, Neet PG Coaching, FMGE, USMLE
Question: 88
Which of the following is not an indication of DPL in abdominal trauma?
B FAST is inconclusive
D CT scanning is not
available with equivocal physical
exam findings like lap-belt sign,
injuries to adjacent structures
such as the lower ribs.
Explanation:
Question: 89
A 45 year old gentleman has undergone truncal vagotomy and
pyloroplasty for bleeding duodenal ulcer seven years ago. Now he has
intractable recurrent symptoms of peptic ulcer. All of the following
suggest the diagnosis of Zollinger Ellison syndrome, except:
https://emedicoz.com/testresult/683510s14692529 75/159
6/9/24, 12:51 PM Neet PG Preparation, Neet PG Coaching, FMGE, USMLE
Explanation:
• A BAO of greater than 15 mEq/h and even as high as 150 mEq/h is indicative
of gastrinoma.
In patients with intermediate gastrin levels (150-1000 pg/mL) and acid secretion,
the secretin stimulation test is helpful. Intravenous secretin (2 U/kg) raises the
serum gastrin levels to higher than 200 pg/mL within 2 minutes and, virtually
always, within 10 minutes in patients with gastrinomas.
Question: 90
Which of the following is true about carcinoma colon
https://emedicoz.com/testresult/683510s14692529 76/159
6/9/24, 12:51 PM Neet PG Preparation, Neet PG Coaching, FMGE, USMLE
Explanation:
Question: 91
A 50 year old male patient, an alcoholic and smoker presents with a 3
hour history of severe retrosternal chest pain and increasing shortness of
breath. He started having this pain while eating following an episode of
forceful vomiting, which was constant and radiated to the back and
interscapular region. He was a known hypertensive. On examination, he
was cold and clammy with a heart rate of 130/min, and a BP of 80/40
mmHg. JVP was normal. All peripheral pulses were present and equal.
Breath sounds were decreased at the left lung base and chest x-ray
showed left pleural effusion. What is the most likely diagnosis?
https://emedicoz.com/testresult/683510s14692529 77/159
6/9/24, 12:51 PM Neet PG Preparation, Neet PG Coaching, FMGE, USMLE
D Acute pulmonary
embolism
Explanation:
Question: 92
Which of the following is not an operation for Hirschsprung’s disease?
A Soave
B Rehbein
C Delorme
D Boley
Explanation:
https://emedicoz.com/testresult/683510s14692529 78/159
6/9/24, 12:51 PM Neet PG Preparation, Neet PG Coaching, FMGE, USMLE
Question: 93
Which of the following disappears by itself by the age of 7-8 years?
A Salmon’s patch.
B Strawberry angioma.
D Plexiform angioma.
Explanation:
Question: 94
Most common flap used after Radical neck dissection is?
https://emedicoz.com/testresult/683510s14692529 79/159
6/9/24, 12:51 PM Neet PG Preparation, Neet PG Coaching, FMGE, USMLE
A Deltopectoral flap
B Trapezius flap
Explanation:
Question: 95
Maximum strength is a wound is achieved at?
A 6th week
B 8th week
https://emedicoz.com/testresult/683510s14692529 80/159
6/9/24, 12:51 PM Neet PG Preparation, Neet PG Coaching, FMGE, USMLE
C 10th week
D 12th Week
Explanation:
Correct Answer (D) Scar: Strength 1 week: 3% of normal skin 3rd week: 20% 12th
week: 80% (Max) A wound never gets its 100% strength back
Question: 96
Tumors of anterior mediastinum include the following except
A Thymoma
B Lymphoma
D Schwanoma
Explanation:
https://emedicoz.com/testresult/683510s14692529 81/159
6/9/24, 12:51 PM Neet PG Preparation, Neet PG Coaching, FMGE, USMLE
Question: 97
Commonest melanoma seen in elderly people on the face is?
A Nodular.
B Superficial spreading.
C Acral Lentigenous.
Explanation:
Correct Answer (D) • Superficial spreading melanoma: Most common type (65-
70%). Most common on the trunk in men and women and on the legs in women
• Nodular melanoma: About 25% are of this type. It is also found in parts of the
body only exposed to the sun. So it is most often found on the chest or back.
Nodular melanomas are often very dark brownish black or black in colour. The
depth of the lesion appears to correlate with the prognosis of the patient, and
nodular melanoma is less often amenable to definitive treatment than is the
superficial spreading variety. • Lentigo maligna melanoma: 10% are this type.
Lentigo MM is most common in elderly people and commonest on the face. This
type of melanoma grows very slowly. • Acral lentiginous melanoma: This type is
https://emedicoz.com/testresult/683510s14692529 82/159
6/9/24, 12:51 PM Neet PG Preparation, Neet PG Coaching, FMGE, USMLE
most commonly found on the palms and soles or around the big toenail. It can
also grow under the nails.
Question: 98
The most common cause of hypercalcemic crisis is:
A Sarcoidosis
B Primary
hyperparathyroidism
C Malignancy
D Renal failure
Explanation:
Treatment:
If diuresis is restricted and does not promise a relevant effect within a few hours,
hemodialysis (calcium free) is the treatment of choice
https://emedicoz.com/testresult/683510s14692529 83/159
6/9/24, 12:51 PM Neet PG Preparation, Neet PG Coaching, FMGE, USMLE
Question: 99
Kalloo 45 years presents with a swelling in the thyroid gland and a
lymphnode in the neck. Aspiration of the node shows amyloid material.
What is the management of choice for this patient
A Hemithyroidectomy with
neck dissection
D Hemithyroidectomy
Explanation:
Question: 100
https://emedicoz.com/testresult/683510s14692529 84/159
6/9/24, 12:51 PM Neet PG Preparation, Neet PG Coaching, FMGE, USMLE
A Atherosclerosis
C Fibromuscular dysplasia
Explanation:
(Fibromuscular dysplasia)
Question: 101
Match the following
https://emedicoz.com/testresult/683510s14692529 85/159
6/9/24, 12:51 PM Neet PG Preparation, Neet PG Coaching, FMGE, USMLE
Explanation:
Question: 102
A drug was tested in the electrophysiology laboratory to determine its
effects on the cardiac action potential in normal ventricular cells. The results
are shown in the diagram. Which of the following drugs does this agent
most resemble?
https://emedicoz.com/testresult/683510s14692529 86/159
6/9/24, 12:51 PM Neet PG Preparation, Neet PG Coaching, FMGE, USMLE
A Ibutilide
B Flecainide
C Mexiletine
D Procainamide
Explanation:
Correct Answer (B) The drug effect shown in the diagram includes slowing of the
upstroke of the AP but no significant change in repolarization or AP duration.
This is most typical of group 1C drugs. Ibutilite- class III anti arrhythmic agent
prolongs the APD Mexiletine – class Ib agent shorts the APD Procainamide –
class Ia prolong the APD
Question: 103
In a study of new diuretics, an investigational drug was given daily for 8 d
and urine output was analyzed. The following data were obtained. Which
mechanism best explains the effects shown on the graph?
https://emedicoz.com/testresult/683510s14692529 87/159
6/9/24, 12:51 PM Neet PG Preparation, Neet PG Coaching, FMGE, USMLE
A Carbonic anhydrase
inhibition
C Blockade of a NaCl
transporter in the distal
convoluted tubule
Explanation:
Correct Answer (C) The graph shows a moderate diuresis, some potassium
wasting, and metabolic alkalosis. This is most consistent with a thiazide diuretic
such as hydrochlorothiazide
Question: 104
Which of the following drugs often causes tachycardia and tremor when
used in asthma?
https://emedicoz.com/testresult/683510s14692529 88/159
6/9/24, 12:51 PM Neet PG Preparation, Neet PG Coaching, FMGE, USMLE
A Albuterol
B Cromolyn sodium
C Ipratropium
D Montelukast
Explanation:
Correct Answer (A) Skeletal muscle tremor is a common adverse effect of beta-
adrenoceptor agonists such as albuterol when used in asthma. Even when
administered via inhalation, this side effect (as well as tachycardia) can occur.
While an antimuscarinic can cause tachycardia, it will not cause tremor, and
ipratropium is not usually absorbed in sufficient quantities to cause systemic
antimuscarinic effects.
Question: 105
Of the following drugs, which has established clinical uses that include
attention deficit hyperkinetic disorder, enuresis, and the management of
chronic pain?
A Bupropion
B Citalopram
C Imipramine
D Risperidone
https://emedicoz.com/testresult/683510s14692529 89/159
6/9/24, 12:51 PM Neet PG Preparation, Neet PG Coaching, FMGE, USMLE
Explanation:
Question: 106
A 24-year-old woman is to be treated with levofloxacin for a urinary tract
infection. A contraindication to the use of the antibiotic in this patient is
B Glucose-6-phosphate
dehydrogenase (G6PD) deficiency
C Gout
D Q-T prolongation
Explanation:
Correct Answer (D) Patients with a history of cardiac irregularities should avoid
certain fluoroquinolones including levofloxacin and moxifloxacin since they are
known to cause QT prolongation.
Question: 107
A 55-year-old woman with type 2 diabetes was going to be started on
metformin. Before initiating therapy, it is important to confirm that the
patient has normal renal function because patients with unrecognized
renal insufficiency who take normal doses of metformin are at increased
risk of which of the following?
https://emedicoz.com/testresult/683510s14692529 90/159
6/9/24, 12:51 PM Neet PG Preparation, Neet PG Coaching, FMGE, USMLE
A Hypoglycemia
B Interstitial nephritis
C Lactic acidosis
D Anaemia
Explanation:
Correct Answer (C) Metformin causes adverse effects of commonly diarrhoea, vit
B12 deficiency anaemia, but rarely in the presence of renal failure, hepatic
failure, cardiorespiratory insufficiency and in chronic alcoholic it may cause lactic
acidosis.
Question: 108
The drug in this patient’s regimen that inhibits posttranslational
modification of viral proteins is
A Raltegravir
B Indinavir
C Lamivudine
D Zidovudine
Explanation:
molecules to form the final structural proteins of the mature virion core.
Raltegravir is the integrase inhibitor, lamivudine and zidovudine are NRTIs.
Question: 109
The selective serotonin reuptake inhibitors fluoxetine and paroxetine are
potent inhibitors of hepatic CYP2D6 drugmetabolizing enzymes. This
action may lead to changes in the intensity of the effects of
A Benztropine
B Codeine
C Gentamicin
D Lithium
Explanation:
Correct Answer (B) Codeine, oxycodone, and hydrocodone are all metabolized
by cytochrome CYP2D6, which can be inhibited by certain SSRIs including
fluoxetine and paroxetine. This may lead to a decreased analgesic effects of
codeine that is normally metabolized in part via CYP2D6 forming the more
active compound morphine
Question: 110
Mavacamten useful in
A Breast cancer
B HOCM
https://emedicoz.com/testresult/683510s14692529 92/159
6/9/24, 12:51 PM Neet PG Preparation, Neet PG Coaching, FMGE, USMLE
C IBD
D COPD
Explanation:
Question: 111
The location and name of receptor for absorption of vitamin B12 is?
C Duodenum, Cubilin
D Ileum, Cubilin
Explanation:
Question: 112
Match the following Morphological changes with corresponding causes
https://emedicoz.com/testresult/683510s14692529 93/159
6/9/24, 12:51 PM Neet PG Preparation, Neet PG Coaching, FMGE, USMLE
Explanation:
Question: 113
The vacutainer used for measuring ESR is
A Green
B Purple
C Black
https://emedicoz.com/testresult/683510s14692529 94/159
6/9/24, 12:51 PM Neet PG Preparation, Neet PG Coaching, FMGE, USMLE
D Red
Explanation:
Correct Answer (C) Black. Contains sodium citrate 1;4 ratio has to be taken
Question: 114
A 28-year young healthy male is donating Single donor platelet for the
first time. By the time the procedure is completed the donor develops
tingling sensation in his lips. Which of the following could have occurred
in him?
A Hypoglycaemia
B Hypocalcemia
C Hypomagnesemia
D Hypokalemia
Explanation:
Correct Answer (B) While donating for SDP the rest of the blood (minus
platelets) will be re infused back to the patient since this blood will be mixed
with citrate the patient will develop hypocalcemia.
Question: 115
A 43-year-old man has complained of mild burning substernal pain
following meals for the past 3 years. Upper GI endoscopy is performed
and biopsies are taken of an erythematous area of the lower esophageal
https://emedicoz.com/testresult/683510s14692529 95/159
6/9/24, 12:51 PM Neet PG Preparation, Neet PG Coaching, FMGE, USMLE
A Dysplasia
B Neoplasia
C Anaplasia
D Metaplasia
Explanation:
Correct Answer (D) Metaplasia is the substitution of one tissue normally found at
a site for another. The esophageal stratified squamous epithelium undergoes
metaplasia in response to the ongoing inflammation from reflux of gastric
contents. This is common in the lower esophagus with gastroesophageal reflux
disease (GERD).
Question: 116
A 1 year infant is having facial twitches seizures and recurrent
sinopulmonary infections. He also had an episode of lung abscess and
meningitis. Which of the following finding is expected in this child?
B Increased IgM
https://emedicoz.com/testresult/683510s14692529 96/159
6/9/24, 12:51 PM Neet PG Preparation, Neet PG Coaching, FMGE, USMLE
Explanation:
Correct Answer (D) This child is have DiGeorge syndrome which is associated
with cleft palate and congenital heart disease.
Question: 117
Which of the following transmission patterns is most consistent with this
patient’s family history?
A Autosomal dominant
B X-linked recessive
C X-linked dominant
D Mitochondrial
Explanation:
https://emedicoz.com/testresult/683510s14692529 97/159
6/9/24, 12:51 PM Neet PG Preparation, Neet PG Coaching, FMGE, USMLE
Question: 118
A young boy is being evaluated for developmental delay, mild autism,
and mental retardation. Physical examination reveals the boy to have
large, everted ears and a long face with a large mandible. He is also
found to have macroorchidism (large testes), DNA analysis showed a
trinucleotide disorder. Which one of the following is the correct regarding
this patient?
D It is a autosomal dominant
disease
Explanation:
Correct Answer (B) This is a fragile X syndrome, which is XLR and there is
anticipation happening in the disease. The maternal grandfather will have less
repeats
Question: 119
Which of the following is an intermediate grade vascular neoplasm?
https://emedicoz.com/testresult/683510s14692529 98/159
6/9/24, 12:51 PM Neet PG Preparation, Neet PG Coaching, FMGE, USMLE
A Capillary hemangioma
B Kaposi Sarcoma
C Angiosarcoma
D Glomus tumor
Explanation:
Question: 120
Histological examination of lung from a 1-day old neonate who died is as
shown in the image. What is the likely pathogenesis in this neonate causing
such disease?
A Endothelial injury
B Reduced surfactant
https://emedicoz.com/testresult/683510s14692529 99/159
6/9/24, 12:51 PM Neet PG Preparation, Neet PG Coaching, FMGE, USMLE
C Infection by tuberculosis
D Viral pneumonia
Explanation:
Correct Answer (B) This is histology showing hyaline membranes thus the
answer.
Question: 121
Biopsy from lung mass in a patient is as follows which of the following is
most likely clinical presentation of this patient?
A A middle-aged woman
who has never smoked
https://emedicoz.com/testresult/683510s14692529 100/159
6/9/24, 12:51 PM Neet PG Preparation, Neet PG Coaching, FMGE, USMLE
Explanation:
Correct Answer (C) The biopsy is suggestive of a small cell carcinoma option C is
the one with mets to brain which is a frequent manifestation of small cell
carcinoma
Question: 122
Which of the salivary gland tumor occurs exclusively in parotid glands?
A Pleomorphic adenoma
B Warthin Tumor
D Muco epidermoid
carcinoma
Explanation:
Correct Answer (B) Warthin tumor occurs exclusively in parotid gland, and it is
frequently bilateral and multifocal.
Question: 123
True about celiac disease are all except
A Increased intraepithelial
lymphocytes is earliest
https://emedicoz.com/testresult/683510s14692529 101/159
6/9/24, 12:51 PM Neet PG Preparation, Neet PG Coaching, FMGE, USMLE
histological finding
Explanation:
Correct Answer (B) Celiac have increased risk of Enteropathy associated T cell
lymphomas not B cell.
Question: 124
Renal Biopsy from a patient shows proliferation of mesangial cells and
thickened basement membrane on light microscopy. Immunofluroscence
shows granular deposits of C3 only. Which of the following is the likely
diagnosis?
A MPGN type I
B MPGN type II
C Lupus Nephritis
D IgA Nephropathy
https://emedicoz.com/testresult/683510s14692529 102/159
6/9/24, 12:51 PM Neet PG Preparation, Neet PG Coaching, FMGE, USMLE
Explanation:
Question: 125
Which of the following interleukins is responsible for type I
hypersensitivity?
A IL-1
B IL-4
C IL-22
D INF gamma
Explanation:
Correct Answer (B) IL 4,,13 are secreted by Th2 cells which are responsible for
type I HSR.
Question: 126
In a patient with multiple bilateral nasal polyps with x-ray showing
opacity in the para-nasal sinuses. The treatment consists of all of the
following except:
A FESS
B Corticosteroids
https://emedicoz.com/testresult/683510s14692529 103/159
6/9/24, 12:51 PM Neet PG Preparation, Neet PG Coaching, FMGE, USMLE
C Amphotericin B
D Antihistaminics
Explanation:
Correct Answer (C) Amphotericin B Ref: Read the text below Sol: • The clinical
description is suggestive of ethmoidal polyps. • Here there is no role of anti
fungal agent amphotericin B.
Question: 127
Which of the following symptoms is typically seen in an infant with a type
2 glottic web?
A No symptoms
B Mild hoarseness
D Severe hoarseness,
moderate airway obstruction
Explanation:
Correct Answer (C) Hoarse, weak cry, stridor with exertion Ref: Read the text
below Sol. LARYNGEAL WEB/ATRESIA • Anterior congenital laryngeal webs
typically cause aphonia or stridor. Occasionally a minor web will present as
hoarseness in an older child. The Cohen classification of glottic webs is as
follows: • Type 1: <35% obstruction (mild hoarseness) • Type 2: 35% to 50%
obstruction (hoarse, weak cry, stridor with exertion) • Type 3: 50% to 75%
https://emedicoz.com/testresult/683510s14692529 104/159
6/9/24, 12:51 PM Neet PG Preparation, Neet PG Coaching, FMGE, USMLE
Question: 128
Which of the following is applicable to Otosclerosis?
D. Diplacusis
A A, B and C
B B, C and D
C A, B, C and D
D C and D
Explanation:
A, B and C
Sol:
• With increase in speech intensity above a particular level,the word score falls
rather than maintain a plateau as in cochlear type of sensorineural hearing loss
Question: 129
All are true about adductor spasmodic dysphonia except
B Videostroboscopy is a
useful investigation
https://emedicoz.com/testresult/683510s14692529 106/159
6/9/24, 12:51 PM Neet PG Preparation, Neet PG Coaching, FMGE, USMLE
Explanation:
Correct Answer (A) Patient has breathy voice Ref: Read the text below Sol: • In
adductor spasmodic dysphonia, patient has choked voive ( strangulated voice) In
abductor SD, the voice is breathy.
Question: 130
Study the given tracheostomy tube, it is being used for:
A Preventing Aspiration
B Voice
Explanation:
Correct Answer (B) Voice Ref: Read the text below Sol: • Some tubes have single
or multiple fenestrations on the superior curvature of the shaft (see image
below). • Fenestrations permit airflow, which, in addition to air leaking around
https://emedicoz.com/testresult/683510s14692529 107/159
6/9/24, 12:51 PM Neet PG Preparation, Neet PG Coaching, FMGE, USMLE
the tube, allows the patient to phonate and cough more effectively. That these
tubes allow for patient speech is an important feature.
Question: 131
Which of the following structure is marked by number ‘2’ in the given fig:
A Utricle
B Saccule
C Endolymphatic duct
D Semicircular canal
Explanation:
Saccule
Sol:-
https://emedicoz.com/testresult/683510s14692529 108/159
6/9/24, 12:51 PM Neet PG Preparation, Neet PG Coaching, FMGE, USMLE
Question: 132
A patient with right side hearing loss has tuning fork findings as follows..
Rinne is negative on right side, positive on left side. Weber is lateralized
towards left ear. What is the most probable diagnosis?
Explanation:
https://emedicoz.com/testresult/683510s14692529 109/159
6/9/24, 12:51 PM Neet PG Preparation, Neet PG Coaching, FMGE, USMLE
Sol:
Question: 133
A 70 year old male presents with a 3 month history of right pulsatile
tinnitus with hearing loss, unstable gait,and autophany.ECOG
demonstrates a SP/AP ratio of 0.6. A diagnosis of superior semicircular
canal dehiscence is suspected. Which of the following is a good indicator
test that would give support to this diagnosis:
A Otoacoustic emissions
B Tympanometry
D Auditory brainstem
response
Explanation:
Correct Answer (C) Cervical vestibular evoked myogenic potential Ref: Read the
text below Sol: • Superior semicircular canal dehiscence (SSCD) usually
https://emedicoz.com/testresult/683510s14692529 110/159
6/9/24, 12:51 PM Neet PG Preparation, Neet PG Coaching, FMGE, USMLE
Question: 134
Which of the following is not true about physiology of nose
Explanation:
Correct Answer (B) Sinuses are mainly ventilated during inspiratory phase of
respiration Ref.: Read the text below Sol : • The inspiratory and expiratory air
currents pass through middle part of nasal cavity • Nasal cilia beat 10-20 times
per second and mucociliary clearance time is 10 -20 minutes. • Nasal cycle of
rhythmic congestion and decongestion varies every 2.5 to 4 hours • The sinuses
are ventilated during expiratory phase of respiration.
https://emedicoz.com/testresult/683510s14692529 111/159
6/9/24, 12:51 PM Neet PG Preparation, Neet PG Coaching, FMGE, USMLE
Question: 135
A person met with an accident and suffer from middle cranial fossa
fracture. There was a bluish purple colour behind mastoid. What is your
probable diagnosis?
A Battle sign
B Bezold sign
C Laceration
D Aberation
Explanation:
Correct Answer (A) Battle sign Ref: Read the text below Sol: Battle’s sign appears
as a large bruise that extends across the entire backside of your ear, and it may
also extend out to the upper part of your neck.
Question: 136
Which of the following Thalamic nuclei is a involved in memory?
A Ventroposterolateral
nucleus
B Ventroposteromedial
nucles
https://emedicoz.com/testresult/683510s14692529 112/159
6/9/24, 12:51 PM Neet PG Preparation, Neet PG Coaching, FMGE, USMLE
Explanation:
Correct Answer (D) Anterior thalamic nucleus Anterior thalamic nuclei form an
important component of the Papez circuit, which is involved in conversion of
short-term to long-term memory.
Question: 137
A 65-year-old comatose patient presents is diagnosed with diabetic
ketoacidosis. The patient has increased rate and depth of ventilation
(kussmaul’s breathing). The increase in rate and depth of ventilation is
due to stimulation of which of the following?
A Carotid sinus
B Carotid body
C Medullary chemoreceptors
Explanation:
Correct Answer (B) Carotid body Increase in H+/ decrease in pH stimulates the
peripheral chemoreceptors (carotid and aortic bodies) which in turn increase the
rate and depth of ventilation.
Question: 138
‘y descent’ of JVP coincides with which of the following heart sounds?
https://emedicoz.com/testresult/683510s14692529 113/159
6/9/24, 12:51 PM Neet PG Preparation, Neet PG Coaching, FMGE, USMLE
A First
B Second
C Third
D Fourth
Explanation:
Correct Answer (C) Third ‘y descent’ of JVP is due to rapid flow of blood from the
atria into the ventricles during the first rapid filling phase. Rapid inrushing of
blood from the atria into the ventricles during this phase produces the third
heart sound.
Question: 139
A 25-year-old marathon runner is running a marathon on a hot, humid
day. ADH secretion is maximum in this marathon runner. Tubular fluid in
the PCT is which of the following?
A Isotonic
B Hypotonic
C Hypertonic
https://emedicoz.com/testresult/683510s14692529 114/159
6/9/24, 12:51 PM Neet PG Preparation, Neet PG Coaching, FMGE, USMLE
Explanation:
Correct Answer (A) In the PCT tubular fluid is always isotonic and this is
irrespective of ADH secretion.
Question: 140
A 56-year-old man presents with complaints of fatigue and headaches.
During the physical examination, he is found to have a wide pulse
pressure. Which of the following conditions causes the pulse pressure to
increase?
A Decreased arterial
compliance
Explanation:
Question: 141
A 75-year-old patient is to undergo an emergency laparotomy for
subacute bowel obstruction. He suffers from chronic atrial fibrillation and
is on warfarin. The patient’s INR is 2.0 on admission. What would be the
best course of his management?
https://emedicoz.com/testresult/683510s14692529 115/159
6/9/24, 12:51 PM Neet PG Preparation, Neet PG Coaching, FMGE, USMLE
Explanation:
Question: 142
Concerning train-of-four stimulation:
https://emedicoz.com/testresult/683510s14692529 116/159
6/9/24, 12:51 PM Neet PG Preparation, Neet PG Coaching, FMGE, USMLE
C On administration of non-
depolarizing neuromuscular
blocking agent, fade occurs
before the disappearance of the
twitches
Explanation:
Correct Answer (C) • In TOF, the ratio of the fourth to the first twitch is
measured. • Four twitches of 2 Hz each are applied over 2 seconds. • One should
leave a gap of 10 seconds between each TOF. • As the muscle relaxant is
administered, fade is noticed first, followed by the disappearance of the third
then the second and finally by the first twitch. reversal of the neuromuscular
block is easier if the second twitch is visible. • For routine surgical procedures a
TOF count of 1 or 2 is adequate to achieve adequate relaxation during surgery.
Question: 143
Concerning inhalational anaesthetic potency:
A There is inverse
proportionality between oil: gas
partition coefficients and potency
B MAC is directly
proportional to potency
https://emedicoz.com/testresult/683510s14692529 117/159
6/9/24, 12:51 PM Neet PG Preparation, Neet PG Coaching, FMGE, USMLE
Explanation:
Question: 144
Capnography is an essential component of patient monitoring during
general anaesthesia. As such, end tidal carbon dioxide is one of the
minimum recommended standards of monitoring during anaesthesia.
Carbon dioxide concentration may be measured most using which of the
following methods?
A Gas chromatography
B Raman scattering
C Infrared
spectrophotometry
D Mass spectroscopy
https://emedicoz.com/testresult/683510s14692529 118/159
6/9/24, 12:51 PM Neet PG Preparation, Neet PG Coaching, FMGE, USMLE
Explanation:
Correct Answer (C) Most common method used to monitor end tidal CO2
concentration is Infrared spectroscopy.
Question: 145
The following questions concern intralipid and its role in the treatment of
local anaesthetic toxicity.
Explanation:
Correct Answer (D) • The initial loading dose is 1.5 ml.kg–1. • The infusion dose
thereafter is 0.25 ml.kg–1.min–1. • Propofol, although presented in lipid
emulsion, is not an acceptable substitute in the treatment of local anaesthetic
toxicity because of its cardio-depressant effects. • Intralipid can be used in peri
arrest situations, but if used in cardiac arrest, CPR may have to be prolonged.
https://emedicoz.com/testresult/683510s14692529 119/159
6/9/24, 12:51 PM Neet PG Preparation, Neet PG Coaching, FMGE, USMLE
Question: 146
A 11 month old child presented with recurrent abdominal pain. On
examination a mass can be felt in lumbar region. A barium enema study
was done. What is the likely diagnosis?
A Duodenal atresia
B Intussusception
C Malrotation
D Volvulus
Explanation:
Correct Answer (B) Look for the claw sign in the barium enema image which is
pointing to the diagnosis.
Question: 147
A delayed intravenous urogram of the patient is shown. What is the most
likely diagnosis?
https://emedicoz.com/testresult/683510s14692529 120/159
6/9/24, 12:51 PM Neet PG Preparation, Neet PG Coaching, FMGE, USMLE
A Staghorn calculus
B Putty Kidney
C Pelviuretric junction
obstruction
D Renal Cyst
Explanation:
Correct Answer (C) PUJ obstruction as it is an delayed IVU fil not a plain film.
Question: 148
A 35 year old female presented with fever, expectoration and difficulty
breathing. HRCT of chest was performed and shown. What is the likely
diagnosis?
https://emedicoz.com/testresult/683510s14692529 121/159
6/9/24, 12:51 PM Neet PG Preparation, Neet PG Coaching, FMGE, USMLE
A Mediastinal mass
B Pleural effusion
C Diaphragmatic hernia
Explanation:
Question: 149
Identify the diagnosis
https://emedicoz.com/testresult/683510s14692529 122/159
6/9/24, 12:51 PM Neet PG Preparation, Neet PG Coaching, FMGE, USMLE
A Multiple enchondromas
B Multiple exostosis
C Brown tumors
D Polyostotic fibrous
dysplasia
Explanation:
Question: 150
Not true about signal intensities on MRI
B CSF hyperintense on T2 WI
https://emedicoz.com/testresult/683510s14692529 123/159
6/9/24, 12:51 PM Neet PG Preparation, Neet PG Coaching, FMGE, USMLE
C Melanoma hyperintense on
T1 WI
D Fat hypointense on T1 WI
Explanation:
Correct Answer (D) Fat is hyperintense on T1WI Rest all are correct
Question: 151
The fracture shown in the picture is called as ?
A Diastatic fracture
B Motorcyclist fracture
C Pond fracture
D Depressed fracture
Explanation:
https://emedicoz.com/testresult/683510s14692529 124/159
6/9/24, 12:51 PM Neet PG Preparation, Neet PG Coaching, FMGE, USMLE
Question: 152
Telephono is ?
A Method of recording
evidence
C Torture methodology
D Criminal abortion
technique
Explanation:
Question: 153
CONSIDER THE STATEMENTS: STATEMENT A- Bhang, Ganja and Charas
are products of poppy capsule. STATEMENT B- Their use is characterized
by frenzied desire of committing murders.
https://emedicoz.com/testresult/683510s14692529 125/159
6/9/24, 12:51 PM Neet PG Preparation, Neet PG Coaching, FMGE, USMLE
Explanation:
Question: 154
The stain shown in the picture, is present On the undergarment of a raped
female. It glows in UV Light. This property is due to ?
A Reduced hemoglobin in
stain
B Peroxidase enzyme
C Choline
https://emedicoz.com/testresult/683510s14692529 126/159
6/9/24, 12:51 PM Neet PG Preparation, Neet PG Coaching, FMGE, USMLE
Explanation:
Question: 155
Leading question to a witness in the court of law can be asked ?
B Cross examination
D Both B and C
Explanation:
Question: 156
A 7 year old boy is brought by parents to Ortho OPD with chief complains
of dull aching pain and inability to squat. His problems started with
intense pain in the right hip two weeks ago for which he was treated
conservatively and symptomatically. His blood investigations show raised
ESR with other counts almost normal. X rays were done which showed
osteopenia and mild erosions around the hip joint mainly towards the
acetabular side. Which of the following is the correct diagnosis ?
https://emedicoz.com/testresult/683510s14692529 127/159
6/9/24, 12:51 PM Neet PG Preparation, Neet PG Coaching, FMGE, USMLE
A Perthes’ disease
B Septic Arthritis
C Tuberculosis hip
Explanation:
Question: 157
Match the following types of physeal injuries according to the Salter Harris
Classification :
https://emedicoz.com/testresult/683510s14692529 128/159
6/9/24, 12:51 PM Neet PG Preparation, Neet PG Coaching, FMGE, USMLE
Explanation:
Question: 158
All of the following statements are TRUE , except :
A Kyphoplasty incorporates
injection of PMMA (
Polymethylene methacrylate ) into
the collapsed vertebral body via
balloon technique
https://emedicoz.com/testresult/683510s14692529 129/159
6/9/24, 12:51 PM Neet PG Preparation, Neet PG Coaching, FMGE, USMLE
C Cementing whether
vertebroplasty or kyphoplasty can
be done in tubercular disease
process induced collapsed
vertebral bodies
Explanation:
Question: 159
Which of the following Implant terminology doesn't match with its
Mechanism of Action?
C Ilizarov's Fixator:
Distraction Histiogenesis
https://emedicoz.com/testresult/683510s14692529 130/159
6/9/24, 12:51 PM Neet PG Preparation, Neet PG Coaching, FMGE, USMLE
Explanation:
Question: 160
Identify ?
A Cock-up splint
B Knuckle-bender splint
C Volkmann’s splint
D Crammer-wire splint
Explanation:
Question: 161
https://emedicoz.com/testresult/683510s14692529 131/159
6/9/24, 12:51 PM Neet PG Preparation, Neet PG Coaching, FMGE, USMLE
Study the image given below and identify the embryological basis of the
condition shown:
A Failure of closure of
anterior neuropore
B Failure of closure of
posterior neuropore
Explanation:
Question: 162
Patient is brought to emergency after road traffic accident with multiple
fractures of humerus. On examination he is unable to flex and supinate
forearm. Also there is loss of sensations on lateral aspect of forearm.
Which of the following Nerve is injured?
https://emedicoz.com/testresult/683510s14692529 132/159
6/9/24, 12:51 PM Neet PG Preparation, Neet PG Coaching, FMGE, USMLE
A Ulnar
B Median
C Radial
D Musculocutaneous
Explanation:
Question: 163
Aneurysm of which artery causes visual defects by compressing at optic
chiasma ?
A Anterior cerebral
B Middle cerebral
C Anterior Choroidal
D Anterior communicating
Explanation:
https://emedicoz.com/testresult/683510s14692529 133/159
6/9/24, 12:51 PM Neet PG Preparation, Neet PG Coaching, FMGE, USMLE
Question: 164
A pediatrician noted that some green colored liquid matter is coming out
from the umbilicus of a newborn baby (fecal fistula) admitted in NICU.
What is the embryological basis of this condition:
A Persistence of entire
vitello- intestinal duct
B Persistence of a portion of
vitello-intestinal duct
C Vitello-intestinal duct
replaced by a fibrous band
D Complete obliteration of
vitello- intestinal duct
Explanation:
Question: 165
Identify the type of joint discussed in the given X-ray of pelvis:
https://emedicoz.com/testresult/683510s14692529 134/159
6/9/24, 12:51 PM Neet PG Preparation, Neet PG Coaching, FMGE, USMLE
A Fibrous
B Pivot synovial
C Primary cartilaginous
D Secondary cartilaginous
Explanation:
Question: 166
Which of the following muscles is not cut in episiotomy?
A Bulbospongiosus
B Levator ani
C Transverse perinei
D Ischiococcygeus
Explanation:
Correct Answer (D) Ischiococcygeus The layers cut in episiotomy are vaginal
mucosa, perineal skin, bulbospongiosus, levator ani, superficial and deep
transverse perinei. The anesthesia used is local anesthesia. Suturing is done by
absorbable suture material like polyglactin (rapid vicryl).
https://emedicoz.com/testresult/683510s14692529 135/159
6/9/24, 12:51 PM Neet PG Preparation, Neet PG Coaching, FMGE, USMLE
Question: 167
Which of the following is not the correct pair?
A Ectopic pregnancy –
Methotrexate
D Trichomoniasis –
metronidazole
Explanation:
Question: 168
Primigravida with 41 weeks of gestation has cephalic presentation, no
CPD, no fetal distress. Her cervix is soft, mid-posed, 80% effaced, non-
dilated and station of the fetus is +1. Calculate the Bishop’s score of this
patient:
A 7
B 8
C 9
https://emedicoz.com/testresult/683510s14692529 136/159
6/9/24, 12:51 PM Neet PG Preparation, Neet PG Coaching, FMGE, USMLE
D 10
Explanation:
Question: 169
Which of the following clinical/investigational scenario does not match
with ovulatory cycle?
A Endometrium shows
subnuclear vacuolation
https://emedicoz.com/testresult/683510s14692529 137/159
6/9/24, 12:51 PM Neet PG Preparation, Neet PG Coaching, FMGE, USMLE
Explanation:
Correct Answer (B) Cervical mucous shows ferning pattern on day 22 Ferning is
seen due to salts in the cervical mucous. It is due to estrogenic activity. It is seen
in normal ovulatory patients but between 8th to 18th day of cycle. If there is no
loss of ferning then it means that progesterone has not appeared. This would be
consistent with anovulatory cycle.
Question: 170
A 9 years old girl has developed a visible and palpable breast tissue. She
has scanty, coarse and terminal hair growth on labia majora as well as on
mons pubis. What is the Tanner stage of her puberty?
Explanation:
Correct Answer (A) Tanner 2 for breast development and Tanner 3 for pubic
growth
https://emedicoz.com/testresult/683510s14692529 138/159
6/9/24, 12:51 PM Neet PG Preparation, Neet PG Coaching, FMGE, USMLE
Question: 171
Which one of the following is a part of AMTSL guideline?
A Uterine massage
B Crede’s method
C Inj. Carbetocin
D Inj. Carboprost
Explanation:
Correct Answer (C) Inj. Carbetocin Carbetocin is a long acting, heat stable
oxytocin. It is approved for prevention of PPH in active management of the third
stage of labour. The dose is 100mcg iv
Question: 172
Which one of the following cases must undergo early cord clamping?
B APGAR 1 at 1 minute
Explanation:
https://emedicoz.com/testresult/683510s14692529 139/159
6/9/24, 12:51 PM Neet PG Preparation, Neet PG Coaching, FMGE, USMLE
Correct Answer (B) APGAR 1 at 1 minute HIV is not an indication of early cord
clamping since the advantages of delayed cord clamping are far more than the
risks. Birth asphyxia is an indication of early clamping for the ease of neonatal
resuscitation.
Question: 173
Which one of the following is not true regarding cervical incompetence?
A It is responsible for
recurrent abortions in the first
trimester
B It is treated by cerclage
with absorbable suture material
Explanation:
Correct Answer (D) All of the above Cervical incompetence will not cause first
trimester abortion. It will cause painless second trimester abortion. Suture
material used is non absorbable like mersilene tape. Cerclage in placenta previa
cannot prevent bleeding and it is not recommended for this indication.
Question: 174
The given sonographical picture is seen in:
https://emedicoz.com/testresult/683510s14692529 140/159
6/9/24, 12:51 PM Neet PG Preparation, Neet PG Coaching, FMGE, USMLE
A Ashermann syndrome
Explanation:
Correct Answer (B) Normal endometrial phase in midcycle The USG shows triple
line endometrium made up of 3 hyperechoic lines. These lines are by Zona
basalis and Zona compacta. This is due to estrogenic proliferation which is seen
in midcycle.
Question: 175
A 23 years old primiparous patient has not got her menses since last 3
months. Her Sr. FSH is 6 IU/L. which of the following is the likely cause?
A New pregnancy
https://emedicoz.com/testresult/683510s14692529 141/159
6/9/24, 12:51 PM Neet PG Preparation, Neet PG Coaching, FMGE, USMLE
B Endometrial damage
C Pituitary damage
D Functional hypothalamic
amenorrhoea
Explanation:
Correct Answer (B) Endometrial damage Normal Sr. FSH is 1 to 10 IU/L. In all
other cases, there will be very low Sr. FSH. In a case of endometrial damage; i.e.
Ashermann’s syndrome, there will be normogonadotropic normogonadism
amenorrhoea. In this case the value of FSH is normal.
Question: 176
Which of the following is an incorrect match?
D Ovarian ectopic-----
STUDIFORD criteria
Explanation:
https://emedicoz.com/testresult/683510s14692529 142/159
6/9/24, 12:51 PM Neet PG Preparation, Neet PG Coaching, FMGE, USMLE
Question: 177
Which one of the following is not suitable for treatment with
Methotrexate therapy?
A Heterotrophic ectopic
B GTN
C Ovarian choriocarcinoma
Explanation:
Question: 178
Which of the following is the most accurate method of gestational age
evaluation?
A Naegle’s formula
https://emedicoz.com/testresult/683510s14692529 143/159
6/9/24, 12:51 PM Neet PG Preparation, Neet PG Coaching, FMGE, USMLE
third trimester
Explanation:
Correct Answer (D) CRL evaluation in first trimester The error in the evaluation of
CRL is minimum amongst all the given options. It is just 3-5 days and hence CRL
of 7-9 weeks is the most accurate parameter for evaluation of the gestational
age.
Question: 179
A primigravida with 39 weeks of gestation has cephalic presentation with
head 5/5th palpable abdominally. Her fundal height is 32 cm. Calculate
the fetal weight.:
A 3.1 kg
B 3.25 kg
C 3.5 kg
D 2.9 kg
Explanation:
Correct Answer (A) 3.1 kg This is calculated by Johnson’s formula. It is for the
estimation of fetal weight in gram. It is (FUNDAL HEIGHT – 12) x 155. If the head
https://emedicoz.com/testresult/683510s14692529 144/159
6/9/24, 12:51 PM Neet PG Preparation, Neet PG Coaching, FMGE, USMLE
is engaged then replace 12 by 11. In this case the head is not engaged.
Question: 180
A primigravida with 39 weeks of gestation with normal clinical pelvimetry
and cephalic presentation with average sized baby is in active labour. Her
FHR is 145 beats per minute. On PV examination, her cervix is fully
dilated, fully effaced, station is +3, membranes are absent but the occiput
is in right occipito posterior position. Her uterine contractions are
3/10’/45”. She is in second stage of labour since last 1 hour. What is the
next line of management?
Explanation:
Correct Answer (D) Wait and watch Second stage of labour can last for maximum
3 hours in nulliparous patient and 2 hours for multiparous patient. Fetal
malposition can get corrected so if the labour is progressing well then, we must
do expectant management.
Question: 181
Which one of the following is not correct regarding Down’s syndrome
screening at 18 weeks?
https://emedicoz.com/testresult/683510s14692529 145/159
6/9/24, 12:51 PM Neet PG Preparation, Neet PG Coaching, FMGE, USMLE
B Increased PAPP-A
C Increased inhibin A
D Decreased AFP
Explanation:
Correct Answer (B) Increased PAPP-A PAPP-A is the test done in first trimester in
DUAL marker test which is done between 11 to 13 weeks 6 days.
Question: 182
Identify the following tubal ligation technique:
A Uchida method
B Irving method
C Madlener method
D Pomeroy method
https://emedicoz.com/testresult/683510s14692529 146/159
6/9/24, 12:51 PM Neet PG Preparation, Neet PG Coaching, FMGE, USMLE
Explanation:
Correct Answer (A) Uchida method In this method, the tubal serosa is opened
and then after ligating and cutting the tube, we bury the medial stump under
tubal serosa.
Question: 183
Which of the following is not a mechanism of labour for the given
contraception?
A It causes endometrial
atrophy
C It inhibits ovulation
Explanation:
https://emedicoz.com/testresult/683510s14692529 147/159
6/9/24, 12:51 PM Neet PG Preparation, Neet PG Coaching, FMGE, USMLE
Correct Answer (C) It inhibits ovulation This is MIRENA, it is LNG IUS. It releases
20 mcg of LNG per day. This dose level is not responsible for inhibition of LH
and hence ovulation is not inhibited. It acts of the endometrium level to cause
atrophy and on cervix by making the mucous thick.
Question: 184
Identify the type of fibroid according to FIGO classification?
A FIGO 1
B FIGO 0
C FIGO 3
D FIGO 2
Explanation:
FIGO 1
https://emedicoz.com/testresult/683510s14692529 148/159
6/9/24, 12:51 PM Neet PG Preparation, Neet PG Coaching, FMGE, USMLE
Question: 185
Identify the following placental abnormality:
A Velamentous placenta
B Succenturate placenta
C Circumvallate placenta
D Battledore placenta
Explanation:
Correct Answer (C) Circumvallate placenta There is smaller chorionic plate with
non-utilized ring of decidua basalis. Placenta is surrounded by raised
https://emedicoz.com/testresult/683510s14692529 149/159
6/9/24, 12:51 PM Neet PG Preparation, Neet PG Coaching, FMGE, USMLE
Question: 186
Which of the following condition is not seen in an infant of gestational
diabetic mother?
A Neonatal hyperglycemia
C Polycythemia
D A and B
Explanation:
Correct Answer (D) A and B Neonate will suffer from hypoglycemia due to
excessive insulin in neonate. GDM patients do not have high risk of congenital
anomalies since organogenesis period is not involved.
Question: 187
The likely explanation for the given CTG finding is?
https://emedicoz.com/testresult/683510s14692529 150/159
6/9/24, 12:51 PM Neet PG Preparation, Neet PG Coaching, FMGE, USMLE
A Head compression in
labour
B Cord compression in
labour
C Uteroplacental insufficiency
D Fetal anemia
Explanation:
Question: 188
The most common cause of female pseudo hermaphroditism is?
A CAH
B Adrenal tumour
D AIS
Explanation:
CAH
Question: 189
Which one of the following is not done for the diagnosis of Vesicovaginal
fistula?
C Cystoscopy
D IVP
Explanation:
Correct Answer (B) Q tip cotton swab test Q tip cotton swab test is done for
Stress urinary incontinence. The other tests are done in VVF.
Question: 190
https://emedicoz.com/testresult/683510s14692529 152/159
6/9/24, 12:51 PM Neet PG Preparation, Neet PG Coaching, FMGE, USMLE
B p53 mutation
Explanation:
Correct Answer (A) Obese patient with estrogen excess Most of the endometrial
cancer patient are type 1 cancer patients. These patients have typical estrogenic
excess. These patients are obese and comparatively younger than type 2 cancer
patients.
Question: 191
Non-functional plasma enzyme area all except
A GGT
B Alkaline phosphatase
C Acid phosphatase
D Lipoprotein lipase
https://emedicoz.com/testresult/683510s14692529 153/159
6/9/24, 12:51 PM Neet PG Preparation, Neet PG Coaching, FMGE, USMLE
Explanation:
Question: 192
Hemoglobin and myoglobin are structurally similar in
A Primary structure
B Secondary structure
D Tertiary structure
Explanation:
Question: 193
Methylation of cytidine residues of DNA will cause
A No change
B Mutation
https://emedicoz.com/testresult/683510s14692529 154/159
6/9/24, 12:51 PM Neet PG Preparation, Neet PG Coaching, FMGE, USMLE
Explanation:
Question: 194
If tyrosine level is normal in blood without any external supplement,
deficiency of which of the following is ruled out
A Histidine
B Isoleucine
C Tryptophan
D Phenylalanine
Explanation:
Question: 195
Acetyl CoA obtained from diet in well fed state is least used in the
synthesis of A. Citrate B. Cholesterol C. Palmitoyl CoA D. Acetoacetate
A A and B
https://emedicoz.com/testresult/683510s14692529 155/159
6/9/24, 12:51 PM Neet PG Preparation, Neet PG Coaching, FMGE, USMLE
B C and D
C A, B and C
D Only D
Explanation:
Question: 196
Non scarring alopecia is seen in all except:
A Telogen effluvium
B Androgenetic alopecia
C Alopecia areata
Explanation:
Question: 197
Which of the following is a petechial rash?
https://emedicoz.com/testresult/683510s14692529 156/159
6/9/24, 12:51 PM Neet PG Preparation, Neet PG Coaching, FMGE, USMLE
B Dengue
C Roseola
Explanation:
Correct Answer (B) Dengue Dengue hemorrhagic fever causes petechiae. Hand
foot mouth disease is a blister. Roseola causes red, macular rashes.
Question: 198
A 25-year-old patient presented to STD-OPD with a papular and
umbilicated lesion. Henderson-peterson bodies were seen on histology. The
image shown is. The organism responsible is:
A HPV
B Molluscum
https://emedicoz.com/testresult/683510s14692529 157/159
6/9/24, 12:51 PM Neet PG Preparation, Neet PG Coaching, FMGE, USMLE
C Herpes
D Scabies
Explanation:
Correct Answer (B) Molluscum The image shows molluscum bodies. Molluscum
is a transmissible STD caused by pox virus.
Question: 199
A patient presents with lymphatic dissemination of his disease as shown in
the picture. All the following can cause this, except:
A Nocardia infection
B Staphylococcus infection
C Sporothrix infection
D Mycobacterium marinum
infection
https://emedicoz.com/testresult/683510s14692529 158/159
6/9/24, 12:51 PM Neet PG Preparation, Neet PG Coaching, FMGE, USMLE
Explanation:
Question: 200
The drug of choice for type 2 lepra reaction in pregnancy is:
A Steroid
B Thalidomide
C Chloroquine
D Dapsone
Explanation:
https://emedicoz.com/testresult/683510s14692529 159/159